Medbullets PPT1
A 35-year-old woman presents with joint stiffness and pain of the hands and wrists for the last two months. Her symptoms are most severe in the morning and improve in the afternoon. She reports an unintentional loss of 15 pounds over the course of 3 months as well as fatigue. On physical exam, there is swelling and tenderness at the metacarpophalangeal and proximal interphalangeal joints.
Rheumatoid Arthritis(Snapshot)
A 20-year-old woman presents to her primary care physician's office for discolored fingers when in the cold. She reports that this is not painful, but the sight of her blue fingers bothers her. She reports that it becomes white first, blue, and finally red when she rewarms her hands. Although she has tried wearing gloves, she still reports having this discoloration. She has no significant past medical history. Her physician prescribes her nifedipine.
Raynaud Phenomenon (snapshot)
A 50-year-old female presents to clinic complaining of joint pain. She reports stiffness is her knees, wrists, and hands, that is worst in the morning. Her symptoms have been present for many years. On physical exam, her vital signs are stable, and she has tenderness when her knees, hands, and wrists are articulated. You closely examine her hands (Figure A). Serum rheumatoid factor is positive. Which of the following is true regarding this patient's disease? A) High titers of rheumatoid factor are usually associated with a better prognosis B) Exercise is discouraged as it may worsen range of motion C) The patient is at increased risk of developing osteopenia and osteoporosis D) Opioids are the drugs of choice for pain control in this disease E) Disease modifying drugs such as methotrexate should only be started one year after diagnosis
Rheumatoid arthritis puts patients at an increased risk of developing osteopenia and osteoporosis secondary to corticosteroid use, inactivity, and the disease process itself. Rheumatoid arthritis is a chronic inflammatory autoimmune disease that affects the synovium of the joints and results in cartilage and bone destruction. Key clinical features are a symmetrical inflammatory polyarthritis that may involve any joint except the DIP joints. Most commonly, the hands (PIP and MCP), wrists, knees, ankles, elbows, hips, and shoulders are affected. Treatment involves symptom management with NSAIDs and low dose corticosteroids as well as disease modifying drugs such as methotrexate. Studies have shown an increased risk of bone loss and fracture in individuals with rheumatoid arthritis and a greater risk of osteoporosis, partially iatrogenically caused by glucocorticoid use as well as joint inactivity due to pain. Bone loss is greatest in areas surrounding the affected joints. Rindfleisch and Muller discuss diagnosis and management of rheumatoid arthritis. Prevalence is estimated to be nearly 1% worldwide with women affected twice as often as men. Twenty to thirty percent of patients diagnosed will become work-disabled within two to three years of diagnosis. Diagnosis is mainly clinical with no single lab test confirmatory. Several new disease modifying drugs have become available including TNF inhibitors. Ono et al. discuss secondary osteoporosis and bone loss in rheumatoid arthritis. In rheumatoid arthritis, osteoclasts are activated by abnormal immune conditions accompanied by chronic inflammation resulting in osteoporosis. Decreased bone mineral density and bone quality may expose patients to an increased risk of fracture. The risk of fracture is best accomplished by controlling the underlying inflammation in the disease and by promoting weight-bearing activities. Image A depicts ulnar deviation of the MCP joints. Illustration A depicts the boutonniere deformity of the PIP joints. Illustration B depicts swan-neck deformity of the fingers. Illustration C depicts joint space narrowing and bony erosions / bone displacement in rheumatoid arthritis. Incorrect Answers: Answer 1: High titers of RF are associated with a worsened prognosis. Answer 2: Exercise is encouraged, especially to decrease the risk of osteoporosis. Answer 4: NSAIDs, not opioids, are the first line drug of choice for pain control. Answer 5: Disease modifying drugs should be started at diagnosis to prevent joint destruction.
A 34-year-old man presents to his physician with a complaint of lower back pain and stiffness for the past few months. He describes the pain as being worse in the morning and then gradually improving with activity throughout the day. On physical exam, he has decreased anterior flexion of the lumbar spine. A radiograph of his lumbar spine is obtained and is shown in Figure A. What is the patient's most likely diagnosis? A) Ankylosing spondylitis B) Mechanical lower back pain C) Osteopetrosis D) Psoriatic arthritis E) Reactive arthritis
A, Ankylosing Spondylitis Along with his presentation of lower back pain and stiffness that is worse in the morning and his demographics, this patient's radiograph showing a "bamboo spine" strongly indicates a diagnosis of ankylosing spondylitis. Ankylosing spondylitis is a chronic inflammatory disorder that primarily affects the sacroiliac joint and spine. It most commonly affects males 15-35 years of age, and is strongly associated with the MHC class I serotype, HLA-B27. The disease causes vertebral fusion, which produces symptoms of back pain and stiffness, as well as costovertebral and costosternal joint fusion, which may cause restrictive lung disease due to limited chest wall expansion. Ankylosing spondylitis is a type of seronegative spondyloarthritis (arthritis without rheumatoid factor), along with psoriatic arthritis, inflammatory bowel disease, and reactive arthritis. Figure A is a radiograph showing the characteristic "bamboo spine" seen in patients with ankylosing spondylitis. Radiography of the sacroiliac joints and lumbar spine is the gold standard for evaluating and supporting the diagnosis of ankylosing spondylitis, and shows the characteristic "bamboo spine" indicating vertebral fusion. Incorrect Answers: Answer 2: Mechanical lower back pain does not cause vertebral fusion that is visualized as a bamboo spine on radiograph. In addition, pain and stiffness that is worse in the morning and improves with exercise is more indicative of an inflammatory cause. Answer 3: Osteopetrosis does not cause vertebral fusion that is visualized as a bamboo spine on radiograph. Osteopetrosis is a failure of normal bone resorption that leads to thickened, dense bones that are prone to fracture and seen as diffuse symmetric sclerosis on radiograph. Answer 4: Psoriatic arthritis does not cause vertebral fusion that is visualized as a bamboo spine on radiograph. Psoriatic arthritis is also a seronegative spondyloarthritis and is associated with skin psoriasis and nail lesions. Answer 5: Reactive arthritis does not cause vertebral fusion that is visualized as a bamboo spine on radiograph. Reactive arthritis is also a seronegative spondyloarthritis and is associated with the classic triad of conjunctivitis, urethritis, and arthritis. Bullet Summary: The diagnosis of ankylosing spondylitis can be confirmed with a radiograph of the sacroiliac joints and lower spine showing vertebral fusion (bamboo spine).
A 27-year-old man presents to his primary care provider with low back pain. He has not been to the physician in 6 years. He reports the gradual onset of non-radiating low back pain that started 2 years ago and has progressively worsened over that time. He denies any inciting injury. His pain is worse in the morning and is accompanied by stiffness that improves throughout the day. He has been taking ibuprofen intermittently to treat his pain. He drinks alcohol socially and has a 5 pack-year smoking history. On exam, he is well-appearing and in no acute distress. Palpation of his lumbar vertebral spines elicits no pain. Spine flexion and extension are slightly limited. A radiograph of the patient's pelvis is shown in Figure A. This patient is at an increased risk of developing which of the following? A) Anterior uveitis B) Erythema marginatum C) Erythema nodosum D) Pulmonary arterial hypertension E) Renal insufficiency
A, Ankylosing Spondylitis The patient in this vignette presents with chronic atraumatic low back pain and stiffness with sacroiliac (SI) joint fusion suggestive of ankylosing spondylitis (AS). Patients with AS are at an increased risk of developing uveitis. AS is a seronegative spondyloarthropathy that most commonly presents in men aged 15-35 years of age. It is a chronic inflammatory disorder that primarily affects the SI joints, lumbar spine, and cervical spine. Patients with the HLA-B27 gene are more likely to develop AS. Patients with AS are at an increased risk of developing multiple conditions including psoriasis, inflammatory bowel disease, and anterior uveitis. Figure A demonstrates SI joint fusion, one of the characteristic radiographic findings associated with AS. Incorrect Answers: Answer 2: Erythema marginatum is a type of erythema characterized by pink ring-like non-itchy macules that are most commonly associated with rheumatic fever. Answer 3: Erythema nodosum is a type of erythema characterized by tender red nodules most commonly seen on the anterior legs. Erythema nodosum is often idiopathic but can also be seen in patients with streptococcal infections and autoimmune disorders such as inflammatory bowel disease, Behcet disease, and sarcoidosis. Answer 4: Pulmonary arterial hypertension is characterized by elevated pulmonary artery pressures and can either be primary or secondary to various other conditions. It is associated with certain systemic diseases such as sarcoidosis. Answer 5: Renal insufficiency can occur in the setting of multiple autoimmune diseases including systemic lupus erythematosus. It is not associated with AS. Bullet Summary: Ankylosing spondylitis is a chronic inflammatory disorder of the sacroiliac joints and spine that is associated with anterior uveitis.
A 48-year-old Caucasian female presents to her primary care physician for evaluation of progressive weakness and shortness of breath. She has had progressive shortness of breath over the last year with an intermittent non-productive cough. In addition, she complains of difficulty raising her arms to brush her hair. Her temperature is 99.6°F (37.6°C), pulse is 80/min, blood pressure is 130/85 mmHg, respirations are 18/min, and oxygen saturation is 95% on room air. Neurologic exam shows 4/5 strength in the deltoid muscles bilaterally. Diffuse dry crackles are heard on lung auscultation. A faint erythematous rash is noted on the malar cheeks, periorbital areas, lateral proximal thighs, and the elbows. Which of the following tests is most likely to confirm the diagnosis? A) Muscle biopsy B) CT scan C) Pulmonary function tests D) Serum ANA titer E) Skin biopsy
A, Dermatomyositis / Polymyositis This constellation of proximal muscle weakness and erythematous rash of the peri-orbital areas, extensor surfaces, and proximal lateral thighs is suggestive of dermatomyositis. The most specific diagnostic test is a muscle biopsy showing an inflammatory myopathy with a lymphocytic infiltrate at the periphery of the muscle fascicle (Illustration A). Dermatomyositis is an autoimmune disorder that is 2-3 times more common in females. The classic presentation is proximal muscle weakness in the setting of a characteristic rash. The most commonly seen cutaneous features include: 1. Erythema of the eyelids (heliotrope sign; Illustration B) 2. Thickened, red, scaly plaques of the knuckles and dorsal hands (Gottron's papules; Illustration C) 3. Erythema of the elbows and knees (Gottron's sign; Illustration D) 4. Erythema of the upper back/chest (shawl sign; Illustration E) and other sun-exposed areas Dermatomyositis may be associated with interstitial lung disease, presenting in this case with shortness of breath and dry crackles on lung exam. This patient should be screened for malignancy, especially colon and ovarian cancers which co-occur frequently. Prior to the most accurate test of a muscle biopsy, this patient's workup would typically include serum testing searching for elevated levels of aldolase and CPK secondary to muscle destruction and analysis for elevated anti-Jo-1 (synthetase). This patient would then be treated with steroids and other systemic immunosuppressants. Illustration A shows a muscle biopsy with lymphocytic infiltrate that is predominantly at the periphery of the muscle fascicle consistent with the muscle inflammation seen in dermatomyositis. Illustration B shows the classic heliotrope sign with erythema of the eyelids and some extension on the malar cheeks. Illustration C shows the classic Gottron's papules, which are thickened, red, and scaly plaques that develop over the knuckles and joints of the dorsal hand. Illustration D shows Gottron's sign, which is erythema over the extensor surfaces, especially at the elbows and knees. Illustration E shows the shawl sign, which is characterized by erythema over the upper back and chest (areas of increased sun exposure). Incorrect Answers: Answer 2: A CT scan may be useful for better characterization of this patient's lung disease and screening her for malignancy but would not aid in the diagnosis of dermatomyositis. Answer 3: Pulmonary function tests (PFTs) would be useful in assessing the severity of lung disease in this patient but would not aid in the diagnosis of her underlying disorder. Interstitial lung disease, as seen in dermatomyositis, presents with a restrictive pattern on PFTs, resulting in decreased FEV1 and FVC but a preserved FEV1/FVC ratio > 0.7. Answer 4: A serum anti-nuclear antibody (ANA) is a sensitive but non-specific test often used to rule out systemic lupus erythematosus (it is very unlikely to have lupus with a negative ANA). The ANA may be elevated in dermatomyositis, but would not be the best test to diagnose the disease. Answer 5: A skin biopsy may aid in the diagnosis of dermatomyositis but a muscle biopsy is more specific. Skin biopsies may show interface dermatitis on histologic review - a finding that may be seen in both dermatomyositis and systemic lupus erythematosus. Bullet Summary: Dermatomyositis is an autoimmune disorder that classically presents with proximal extremity muscle weakness, characteristic rashes (Gottron's papules, Gottron's sign, heliotrope sign, and shawl sign), and an increased risk for internal malignancy. Proximal muscle weakness caused by inflammatory myopathy results in elevated serum aldolase and CPK levels. Muscle biopsy is the confirmatory test of choice.
A 27-year-old woman with no past medical history presents to her primary care provider because she has begun to experience color changes in her fingers on both hands in cold temperatures. She reports having had this problem for a few years, but with the weather getting colder this winter she has grown more concerned. She says that when exposed to cold her fingers turn white, blue, and eventually red. When the problem subsides she experiences pain in the affected fingers. She says that wearing gloves helps somewhat, but she continues to experience the problem. Inspection of the digits is negative for ulcerations. Which of the following is the next best step in treatment? A) Amlodipine B) Phenylephrine C) Propranolol D) Sildenafil E) Thoracic sympathectomy
A, Raynaud Phenomenon This patient's history of digital color changes from white to blue to red in response to cold is consistent with primary Raynaud phenomenon, which is best managed with dihydropyridine calcium channel blockers (DHP-CCBs) such as amlodipine. Primary Raynaud phenomenon (RP) refers to vasospasm of the arteries in response to cold temperatures or emotion that leads to a transient decrease in blood flow. This most commonly occurs in the digits, causing them to change color from white to blue to red before returning to normal. Secondary RP refers to vasospasm in the setting of an associated diagnosis, often an autoimmune condition such as scleroderma or CREST. In severe cases of secondary RP, digital ischemia can result in ulcerations, necrosis, and even require amputation. Patients should initially be advised to avoid triggers and to quit smoking. In the absence of improvement, the first-line medical treatment are DHP-CCBs such as amlodipine, which promote arterial smooth muscle relaxation via prevention of calcium influx. Incorrect Answers: Answer 2: Phenylephrine is a direct alpha agonist, which will promote peripheral vasoconstriction and potentially worsen active primary Raynaud phenomenon. Answer 3: Propranolol is a beta-blocker with activity on beta-1 and beta-2 receptors. Loss of beta-2-mediated vasodilation may promote Raynaud phenomenon. Beta-blockers as a class have been associated with Raynaud phenomenon. Answer 4: Sildenafil is a phosphodiesterase-5 inhibitor that prevents degradation of cyclic-GMP, promoting vasodilation. Sildenafil is indicated in patients with Raynaud phenomenon that are refractory to DHP-CCB therapy. Answer 5: Thoracic sympathectomy removes sympathetic tone to the digital arteries, inhibiting vasoconstriction. This procedure is indicated only in patients with severe disease that have failed medical management. Bullet Summary: Dihydropyridine calcium channel blockers are the first-line medical therapy for Raynaud phenomenon.
A 32-year-old man comes into your office because of pain in his right knee, left elbow, and left wrist. It started about a week ago but has particularly localized to his wrist. The patient states that he has 2 sexual partners. He states he has also had some white discharge from his penis with pruritis and pain during urination. His temperature is 97.6°F (36.4°C), blood pressure is 124/84 mmHg, pulse is 80/min, respirations are 12/min, and oxygen saturation is 98% on room air. Physical exam reveals pain upon palpation of the patient's left wrist which also appears erythematous and swollen. What is the best next step in management for this patient? A) Arthrocentesis B) Azithromycin, ceftriaxone, and vancomycin C) Ibuprofen and observation D) Methotrexate E) MRI
A, Reactive Arthritis This patient likely has a sexually transmitted disease (given his urethral discharge and dysuria) and a migratory arthritis that has localized suggesting a diagnosis of reactive arthritis, gonococcal arthritis, or septic arthritis. He should first be managed with arthrocentesis prior to treating his condition. Reactive arthritis, also called Reiter syndrome, is the most common type of inflammatory polyarthritis in young men. It is more common in those with the HLA-B27 genotype and occurs in those with infections with organisms such as Chlamydia trachomatis (though Salmonella, Shigella, and Campylobacter are less frequent causes). It commonly presents with polyarthritis, conjunctivitis, uveitis, and genitourinary symptoms such as dysuria, polyuria, and penile discharge. The differential for a red, hot, tender, and inflamed joint includes gonococcal arthritis or septic arthritis. Prior to treatment, any patient with a red and hot joint should have arthrocentesis and fluid analysis performed and have a swab for gonorrhoae and chlamydia sent off. Subsequently, the patient can be treated with broad-spectrum antibiotics and surgical washout (if indicated). Incorrect Answers: Answer 2: Azithromycin, ceftriaxone, and vancomycin are broad-spectrum antibiotics that cover both for this patient's STD and a septic arthritis. These antibiotics may be started after arthrocentesis and GC/chlamydia testing have been performed. Answer 3: Ibuprofen and observation is appropriate management of osteoarthritis which presents with cool/normal skin in a weight bearing joint typically in an obese patient. Answer 4: Methotrexate is a disease-modifying antirheumatic drug (DMARD)that is indicated for long-term management to improve outcomes and reduce the incidence of permanent joint deformity in rheumatoid arthritis which presents in a middle-aged woman with symmetric joint pain that is worse in the morning and improves throughout the day. Answer 5: MRI is an appropriate diagnostic test for osteomyelitis which may present with focal bone pain, a fever, and an elevated ESR/CRP. It would not offer culture results and a gram stain/crystal analysis necessary to diagnose this patient's possible septic arthritis. Bullet Summary: A red, hot, inflamed, and tender joint is concerning for septic arthritis and must initially be managed with arthrocentesis.
A 27-year-old male presents to his primary care physician complaining of pain with urination and eye redness. He reports that he developed these symptoms approximately one week ago. He also has noticed left knee and right heel pain that started a few days ago. He denies any recent trauma. He had an episode of abdominal pain and diarrhea ten days ago that resolved. He has otherwise felt well. On exam, he walks with a limp and his conjunctivae are erythematous. Laboratory findings are notable for an elevated erythrocyte sedimentation rate (ESR) and elevated C-reactive protein (CRP). Which of the following is most likely associated with this patient's condition? A) HLA-B27 haplotype B) HLA-DR4 haplotype C) Anti-cyclic citrullinated peptide (anti-CCP) antibody D) Anti-centromere antibody E) Rheumatoid factor
A, Reactive Arthritis The most likely diagnosis for this patient is reactive arthritis (Reiter's syndrome) which was likely precipitated by a gastrointestinal infection. Individuals who are HLA-B27 positive are at increased risk of developing reactive arthritis. Reactive arthritis is a seronegative spondyloarthropathy that typically presents after an infection. Reactive arthritis can be precipitated by a variety of pathogens including Chlamydia, Salmonella, Shigella, Campylobacter, Yersinia, and Mycoplasma. Reactive arthritis is characterized by a triad of symptoms including urethritis, uveitis/conjunctivitis, and large-joint arthritis. The diagnosis is often made on the basis of clinical history alone; however, laboratory evaluation can help to confirm the diagnosis. CRP and ESR are often elevated. Importantly, individuals who are HLA-B27 positive are at increased risk of developing reactive arthritis. Incorrect Answers: Answer 2: HLA-DR4 is an HLA haplotype associated with certain autoimmune conditions including rheumatoid arthritis and polymyalgia rheumatica. It is not associated with an increased risk of reactive arthritis. Answer 3: Anti-CCP antibody is associated with rheumatoid arthritis. Rheumatoid arthritis presents with bilateral symmetric arthritis that frequently affects the knees, metacarpal-phalangeal joints, and proximal interphalangeal joints. The pain often improves throughout the day. Given this patient's symptoms of urethritis and conjunctivitis as well as the preceding history of gastrointestinal infection, reactive arthritis is the more likely diagnosis. Answer 4: Anti-centromere antibody is seen in patients with CREST syndrome, an autoimmune disorder characterized by joint calcinosis, Raynaud's phenomenon, esophageal dysfunction, sclerodactyly, and telangiectasias. This patient's history of large joint arthritis, urethritis, and conjunctivitis following a gastrointestinal infection is more consistent with reactive arthritis. Answer 5: Rheumatoid factor is an auto-antibody directed against the Fc portion of the IgG immunoglobulin. It is seen in a variety of "seropositive" autoimmune conditions including rheumatoid arthritis, systemic lupus erythematosus, and Sjogren's syndrome. It is not associated with reactive arthritis, which is one of several conditions known as "seronegative" sponyloarthropathies because they are not associated with rheumatoid factor. Bullet Summary: Reactive arthritis typically presents with urethritis, conjunctivitis, and large-joint arthritis following a gastrointestinal or sexually-transmitted infection. Individuals who are HLA-B27 positive are at increased risk of developing reactive arthritis.
A 55-year-old woman presents to the emergency department with chest pain, shortness of breath, and weakness. She has no known past medical history and generally refuses to see a physician for health issues. Review of systems is notable for chronic, severe gastroesophageal reflux disease and chronic diarrhea. Her temperature is 98.3°F (36.8°C), blood pressure is 177/105 mmHg, pulse is 88/min, respirations are 14/min, and oxygen saturation is 97% on room air. Laboratory values are ordered as seen below. Hemoglobin: 10 g/dL Hematocrit: 30% Leukocyte count: 4,500/mm^3 with normal differential Platelet count: 192,400/mm^3 Serum: Na+: 139 mEq/L Cl-: 101 mEq/L K+: 6.3 mEq/L BUN: 65 mg/dL Glucose: 99 mg/dL Creatinine: 3.1 mg/dL Notably, the patient requires nursing to help her with most tasks such as putting on her gown and manipulating a cup of water given poor mobility of her hands. She also has recurrent episodes of severe hand pain, which self resolve. The patient is given calcium, insulin, and dextrose and started on dialysis. Which of the following is the most appropriate medical therapy for this patient? A) Captopril B) Furosemide C) Hydrochlorothiazide D) Labetalol E) Nifedipine
A, Scleroderma This patient is presenting with a history of undiagnosed scleroderma (Raynaud phenomenon, poor mobility of her hands, chronic reflux disease, and chronic diarrhea) and an acute episode of scleroderma renal crisis (hypertension, hyperkalemia, and an elevated BUN/Cr) warranting medical treatment with captopril. Scleroderma is a systemic connective tissue disease where increased collagen production can cause many different symptoms. Patients can present with smooth and taut skin, poor mobility of their hands, Raynaud disease, diarrhea (from fibrosis and bowel stasis), reflux disease (from dilated esophagus), interstitial lung disease, and acute kidney failure in scleroderma renal crisis. Scleroderma renal crisis presents with hypertension and signs of decreased kidney function including hyperkalemia and an increased BUN/creatinine. Patients will need dialysis and should be started on captopril which is a nephroprotective and mortality-lowering agent in scleroderma renal crisis. Incorrect Answers: Answer 2: Furosemide could be given in hyperkalemia and congestive heart failure; however, it has no mortality benefit in scleroderma renal crisis and is not nephroprotective. Answer 3: Hydrochlorothiazide is a thiazide diuretic that could be used for blood pressure control for hypertension but is not appropriate and is not nephroprotective in scleroderma renal crisis. Answer 4: Labetalol is an alpha- and beta-blocker which could be given in a hypertensive emergency; however, it is much direr to give the nephroprotective agent captopril. Dialysis will lower this patient's blood pressure and her response to dialysis should be monitored before giving an acute blood pressure medication. Answer 5: Nifedipine is a calcium channel blocker that can be given for Raynaud disease (which this patient has given her recurrent vasospastic hand pain); however, it is more important to first start captopril which has mortality lowering effects. The goal is to give the max dose of captopril without causing hypotension thus starting other non-mortality-lowering medications should be held until later. Bullet Summary: Captopril is appropriate medical therapy for scleroderma renal crisis.
A 49-year-old female patient was sent from her primary care physician to the emergency department (ED) for the management of marked hypertension, and acute renal failure. In the ED, she claims that she has fatigue, stiffening in the joints, muscle pain, pain in her fingertips, and paleness in her fingers when she is in a cold environment. She has been having these symptoms for some time. She has noticed that the volume of urine she usually produces is significantly decreased. Vitals are remarkable for a blood pressure of 170/105 mmHg. Physical examination is notable for taut, thickened skin in the face, hands, and fingers (Figure A). Her creatinine is 4.2 mg/dL, from a baseline of 1.2 mg/dL. Which of the following is a property of the medication that will most likely improve survival in this patient's current condition? A) Can decrease mortality in heart failure B) Decreases bradykinin levels C) Decreases circulating renin levels D) Blocker of dihydropyridine calcium channels E) Only decreases preload
A, Scleroderma This patient's physical exam findings and history is highly suggestive of scleroderma renal crisis. Angiotensin converting enzyme inhibitors (ACE-I) is the mainstay of treatment to preserve renal function, and improve survival in these patients. ACE-I have been shown to decrease mortality in patients with heart failure. Scleroderma renal crisis (SRC) is a complication of systemic sclerosis. Patients can present with marked hypertension of abrupt onset, as well as significant rapid impairment of renal function. Pathogenesis of SRC is not well understood. Corticosteroid exposure, especially at high doses, seems to increase the risk of developing SRC. Diffuse skin involvement also seems to represent an important risk factor. Captopril is commonly used to manage these patients. With regards to heart failure, it is important to remember that ACE-I, beta-blockers (i.e., metoprolol, carvedilol), and aldosterone antagonists (i.e., spironolactone) have been shown to decrease mortality in patients with heart failure. Hinchcliff and Varga present the clinical presentation and treatment of organ involvement in patients with scleroderma. Characteristically, patients with SRC present with accelerated hypertension of sudden onset, which may be accompanied with oliguric renal failure and proteinuria, microangiopathic anemia, as well as microscopic hematuria. SRC was the considered mostly fatal before ACE-I were introduced. ACE inhibitors should be promptly used if SRC is suspected. Steen et al. explored the outcome of renal crisis in systemic sclerosis before and after ACE-I availability. This was a prospective cohort study. Their data suggests that ACE-I therapy dramatically improved survival in patients with SRC. Along with improved survival, there is a possibility of successful dialysis discontinuation with ACE-I use. Figure A depicts sclerodactyly, which is characterized by tightening and thickening of the digits. This can lead skin ulceration. Incorrect Answers Answer 2: Decreasing bradykinin levels is incorrect. ACE-I would increase bradykinin levels, due to ACE being involved in converting bradykinin into inactive metabolites. This can lead to angioedema. Answer 3: An increase, not decrease, in renin levels would be expected with ACE-I use. Answer 4: Calcium channel blockers are not considered first-line for the treatment of SRC. They can be added with ACE-I treatment and can be used to manage Raynaud's. Answer 5: Having a decrease only in the preload is incorrect. ATII is a powerful vasoconstrictor. ACE-I decrease both the preload and afterload.
A 27-year-old woman presents to her primary care physician for evaluation of malaise, joint pains, and rash. Over the last 1-2 months, she has felt generally unwell with fatigue and low-grade fever. More recently, she has started to experience joint pains in her hands and noticed a rash over her face after sun exposure. She is otherwise healthy and takes no medications. Examination reveals an erythematous rash with a small amount of underlying edema (Figure A). Which of the following is the most common cardiac manifestation of this patient's underlying condition? A) Pericarditis B) Myocarditis C) Conduction arrhythmias D) Valvular disease E) Coronary artery disease
A, Systemic Lupus Erythematosus This patient presents with a clinical history consistent with systemic lupus erythematosus (SLE). The most common cardiac manifestation of SLE is pericarditis. SLE can affect the heart in numerous ways. Most commonly it causes pericarditis, with a pericardial effusion present in as many as half of patients. Patients are not typically symptomatic from pericarditis, but suggestive history includes substernal, pleuritic chest pain with an audible rub on examination. SLE can also cause valvular disease, most often mitral regurgitation. Large verrucous vegetations (typically mitral, aortic, or tricuspid) in the context of SLE is referred to as Libman-Sacks endocarditis. Snyder et al. discuss the diagnosis and management of acute pericarditis. ECG findings in acute pericarditis include widespread concave ST-segment elevations and depression of the PR-segment. They highlight predictors of severe illness, including high fever, evidence of tamponade or a large effusion, or subacute onset; these factors warrant strong consideration of inpatient admission rather than outpatient treatment. Farzaneh-Far et al. present a series of 200 patients with SLE who underwent extensive cardiac evaluation. They sought to assess the relationship between the presence of antiphospholipid antibodies (APL) and cardiac disease. They found a strong correlation between the presence of high levels of APL and mitral valve disease, including nodules and valvular regurgitation. APL levels did not correlate with altered LV size or function. Figure A shows a typical "butterfly" (or malar) rash in a patient with SLE. This rash is photosensitive, spares the nasolabial folds, and occurs in up to half of patients with SLE. Incorrect Answers: Answer 2: Myocarditis can occur with SLE, and similarly is often asymptomatic, but is less common than pericarditis. Answer 3: Conduction abnormalities typically represent scarring damage from prior myocarditis in SLE. When present, it is often a transient first-degree AV block. Answer 4: Mitral valve disease (especially with resultant regurgitation) is a common cardiac manifestation of SLE, but pericarditis is more common. Mitral valve disease is especially associated with Libman-Sacks endocarditis. Answer 5: Patients with SLE are at markedly increased risk for coronary artery disease, but it is still less common than pericarditis as a cardiac manifestation of SLE.
A 21-year-old man presents to his physician's clinic with lower back pain. He reports the pain is accompanied by stiffness, is worse in the morning, and improves with exercise. He denies any radiation of pain or bowel and bladder incontinence. On physical exam, there is decreased anterior flexion of the lumbar spine. Laboratory testing is positive for HLA-B27. A plain radiograph of the spine is shown.
Ankylosing Spondylitis (Snapshot)
A 21-year-old man presents to his physician's clinic with lower back pain. He reports the pain is accompanied by stiffness, is worse in the morning, and improves with exercise. He denies any radiation of pain or bowel and bladder incontinence. On physical exam, there is decreased anterior flexion of the lumbar spine. Laboratory testing is positive for HLA-B27. A plain radiograph of the spine is shown.
Ankylosing Spondylitis (snapshot)
A 41-year-old female presents to her primary care doctor with worsening muscle pain and weakness. Approximately two months ago, she reports the recent development of having difficulty climbing stairs and raising her arms above her head. Additionally, there was a dull aching pain in her shoulders over the same time period. Her family history is notable for systemic lupus erythematosus in her mother. Physical examination demonstrates symmetric muscle weakness in hip flexion, hip extension, and shoulder abduction. She has red scaly skin overlying the extensor joints of her fingers and has another dermatological finding seen in Figure A. Which of the following is associated with this patient's most likely condition? A) Anti-cyclic citrullinated peptide antibody B) Anti-DNA topoisomerase I antibody C) Perimysial inflammation with infiltration of CD4+ T-cells D) Endomysial inflammation with infiltration of CD8+ T-cells E) Loss of dystrophin expression
C, Polymyositis / Dermatomyositis The most likely diagnosis in this patient is dermatomyositis. Dermatomyositis is characterized by inflammation of the muscle perimysium and CD4+ T-cell infiltration. Dermatomyositis is a systemic autoimmune connective tissue disorder characterized by inflammation of the skin ("dermato") and muscles ("myo"). Skin-related symptoms of dermatomyositis include red patches on the eyelids (heliotrope rash), red scaly skin over the knuckles (Gottron papules), and a diffuse erythematous rash over the neck and shoulders (shawl sign). Patients also commonly present with symmetric proximal muscle weakness primarily affecting the hips and shoulders. This leads to difficulty with climbing stairs, rising from a chair, and raising arms above the head. Dermatomyositis often arises in adult females in their forties and fifties. It can also be seen as a paraneoplastic syndrome in patients with occult malignancies. The tissue damage seen in dermatomyositis is antibody-mediated. Histologically, dermatomyositis is characterized by perimysial inflammation with infiltration of CD4+ T-cells. Figure A demonstrates a heliotrope rash. This rash typically presents as a violaceous eruption on the upper eyelids and is highly characteristic of dermatomyositis. Unlike the malar rash seen in SLE, the heliotrope rash typically affects the eyelids and spares the skin below the eyes. Incorrect Answers: Answer 1: Anti-cyclic citrullinated peptide (CCP) antibody is specific for rheumatoid arthritis (RA). RA is a systemic autoimmune disease that presents with symmetric joint pain and stiffness that improves with use. Subcutaneous nodules can be seen in RA, but the skin findings and proximal muscle weakness in this patient are more consistent with dermatomyositis. Answer 2: Anti-DNA topoisomerase I (anti-scl-70) antibody is elevated in scleroderma. Scleroderma is a systemic fibrosing disease that presents with skin swelling, tightened facial features, and polyarthralgia in addition to gastrointestinal, respiratory, and renal complications. Answer 4: Endomysial inflammation with infiltration of CD8+ T-cells is seen in polymyositis, a disorder that is similar to dermatomyositis. The primary clinical difference between the two disorders is that polymyositis lacks associated skin findings. The main pathophysiologic difference between the two disorders is that the tissue damage in polymyositis is CD8+ T-cell-mediated, while the tissue damage seen in dermatomyositis is antibody-mediated. Answer 5: Loss of dystrophin expression is characteristic of Duchenne muscular dystrophy (DMD). This commonly presents in young male children with proximal muscle weakness, fatigability, and pseudohypertrophy of the calf muscles. DMD is not associated with skin findings. Bullet Summary: Dermatomyositis is a systemic autoimmune disorder typically causing proximal muscle weakness and multiple skin findings. Histologically, muscle tissue in patients with dermatomyositis may demonstrate antibody-mediated perimysial inflammation with CD4+ T cell infiltration.
A 34-year-old woman presents to her primary care physician complaining of weakness and rash for the past 2 months. She says her symptoms had a gradual onset at first, but over the past 2 weeks she has grown more concerned as she now has difficulty going up and down the stairs in her apartment. Her rash is nonpruritic and confined to her knuckles. She denies fever, chills, and changes in bowel habits, cough, or shortness of breath. Up until the onset of her symptoms she had been an avid hiker in upstate New York. Temperature is 98.0°F (36.7°C) blood pressure is 110/72 mmHg, pulse is 88/min, and respirations are 20/min. Physical examination demonstrates 4-/5 strength in the proximal lower extremities. The patient's cutaneous findings are demonstrated in figure A. Laboratory studies will most likely demonstrate the following in this patient? A) Anti-endomysial antibodies B) Anti-histidyl tRNA synthetase antibodies C) Anti-histone antibodies D) Anti-HMG CoA reductase antibodies E) Anti-mitochondrial antibodies
B, Polymyositis / Dermatomyositis This patient's proximal muscle weakness and cutaneous findings consistent with Gottron papules are consistent with a diagnosis of dermatomyositis, which will most likely have anti-histidyl tRNA synthetase antibodies. Dermatomyositis is an autoimmune disease characterized primarily by muscle weakness and dermatologic findings. Weakness in dermatomyositis is proximal and usually painless, and patients often complain of difficulty going up and down stairs, getting up from a seated position, and difficulty lifting objects. The dermatologic findings of dermatomyositis include heliotrope rash, Gottron papules, V-sign, shawl sign, holster sign, and mechanic hands. While a multitude of autoantibodies has been identified in association with dermatomyositis, anti-Jo antibodies (anti-histidyl tRNA synthetase antibodies) are the most common. Figure A demonstrates Gottron papules. Incorrect Answers: Answer 1: Anti-endomysial antibodies are specific for celiac disease, and are not associated with dermatomyositis. Answer 3: Anti-histone antibodies are highly sensative for drug-induced lupus and are not used in the diagnosis of dermatomyositis. Answer 4: Anti-HMG CoA reductase antibodies have been implicated in patients with severe necrotizing statin-mediated myopathy not dermatomyositis. Answer 5: Anti-mitochondrial antibodies are seen in primary biliary cirrhosis not dermatomyositis. Bullet Summary: Anti-histidyl tRNA synthetase antibodies (anti-Jo antibodies) are associated with dermatomyositis.
A 32-year-old woman presents to the primary care clinic complaining of intermittently discolored hands. She states that whenever she becomes stressed, she notices her fingers turn white, blue, and then eventually warm and pink. She denies pain, but endorses "tightness" as well as mild pruritis when they turn pink. The patient also complains of fatigue, but she feels that this is from her new job as a night nurse. Her medical history is significant for panic disorder. She takes sertraline. After running her hands under cold water in the exam room, her fingers turn white, as shown in Figure A. Which of the following is the best management for the patient's symptoms? A) Hydroxyurea B) Nifedipine C) Propranolol D) Tadalafil E) Verapamil
B, Raynaud Phenomenon The patient is presenting with fingers that turn white, blue, and then pink with stress, which is consistent with Raynaud. First-line pharmacologic treatment for Raynaud is a calcium channel blocker, such as nifedipine. Raynaud phenomenon is characterized by discoloration of the extremities as a physiologic response to cold temperatures or emotional stress. Initially, digits turn white (indicating ischemia), blue (indicating hypoxia and cyanosis), and finally pink with re-warming (indicating reperfusion). Raynaud can be idiopathic or associated with conditions such as systemic sclerosis/limited scleroderma, systemic lupus erythematosus, mixed connective tissue disease, and cryoglobulinemia. Conservative management involves avoidance of cold temperatures and smoking cessation. First-line pharmacologic management is dihydropyridine-type calcium channel blockers (i.e., nifedipine, nicardipine, or amlodipine). Figure A shows a picture of fingers that have turned discretely white, which is characteristic of Raynaud phenomenon. Incorrect Answers: Answer 1: Hydroxyurea is used in the treatment of sickle cell disease. Patients can present with painful swelling of the hands and feet, called dactylitis. Dactylitis is not associated with white and blue ischemic color changes. Answer 3: Propranolol is a beta blocker that can actually worsen Raynaud syndrome. Answer 4: Tadalafil is a phosphodiesterase inhibitor that can be used to treat Raynaud. However, it is indicated for patients who are refractory to calcium channel blockers. Answer 5: Verapamil is a nondihydropyridine-type calcium channel blocker that would not be as effective as dihydropyridines for treatment of Raynaud. This is because the dihydropyridines act on vascular smooth muscle. Bullet Summary: First-line pharmacologic treatment of Raynaud phenomenon includes dihydropyridine-type calcium channel blockers.
A 33-year-old Caucasian female presents to her primary care provider for pruritus and shortness of breath. Over the past year, she has experienced mild progressive diffuse pruritus. She also reports that her skin seems "hard" and that it has been harder to move her fingers freely. She initially attributed her symptoms to stress at work as a commercial pilot, but when her symptoms began impacting her ability to fly, she decided to seek treatment. She has a history of major depressive disorder and takes citalopram. She smokes 1 pack per day and drinks socially. Her temperature is 98.6°F (37°C), blood pressure is 148/88 mmHg, pulse is 83/min, and respirations are 21/min. On exam, she appears anxious with increased work of breathing. Dry rales are heard at her lung bases bilaterally. Her fingers appear shiny and do not have wrinkles on the skin folds. A normal S1 and S2 are heard on cardiac auscultation. This patient's condition is most strongly associated with which of the following antibodies? A) Anti-cyclic citrullinated peptide B) Anti-DNA topoisomerase I C) Anti-double-stranded DNA D) Anti-SS-A E) Anti-U1-ribonucleoprotein
B, Scleroderma The patient in this vignette presents with progressive dyspnea and skin hardening suggestive of systemic scleroderma. Scleroderma is associated with anti-DNA topoisomerase I (anti-Scl-70) antibodies. Scleroderma is an autoimmune condition characterized by hardening and induration of the skin and/or other organs due to collagen deposition and fibrosis. Scleroderma can cause a restrictive lung disease that is triggered by elevated TGF-beta secretion by T cells in the lungs. This leads to collagen deposition, vascular fibroproliferation, and chronic inflammation. Clinically, this may present with dyspnea, dry rales, and interstitial fibrosis on lung imaging. As with many other autoimmune diseases, scleroderma is associated with the presence of auto-antibodies, specifically antibodies directed against DNA topoisomerase I. Incorrect Answers: Answer 1: Anti-cyclic citrullinated peptide (anti-CCP) antibodies are specific for rheumatoid arthritis (RA). RA is a systemic autoimmune disease that presents with symmetric joint pain and stiffness that improves with use. Answer 3: Anti-double-stranded DNA (anti-dsDNA) antibodies are specific for systemic lupus erythematosus (SLE). SLE is a systemic autoimmune disease that presents with constitutional symptoms along with variable musculoskeletal, cutaneous, and neurologic symptoms. Answer 4: Anti-SS-A (Anti-Ro) antibodies are specific for Sjogen syndrome. Sjogen syndrome is a systemic autoimmune disease affecting the exocrine glands that presents with sicca symptoms (e.g., decreased saliva production and decreased tear production) and constitutional symptoms. Answer 5: Anti-U1-ribonucleoprotein (anti-U1-RNP) antibodies are specific for mixed connective tissue disease (MCTD). MCTD is a systemic autoimmune disease that presents similarly to SLE, systemic scleroderma, and polymyositis. Bullet Summary: Scleroderma is associated with antibodies directed against DNA topoisomerase I (anti-Scl-70).
A 43-year-old woman presents with complaints of retrosternal burning associated with eating. It has persisted for the past several years but has been getting worse. Her past medical history is unknown and this is her first time seeing a doctor. She states she is otherwise healthy and review of systems is notable for episodic hand pain that is worse in the winter as well as a chronic and severe cough with dyspnea which she attributes to her smoking. Her temperature is 97.7°F (36.5°C), blood pressure is 174/104 mmHg, pulse is 80/min, respirations are 22/min, and oxygen saturation is 92% on room air. Physical exam is notable for a young appearing woman with coarse breath sounds. Laboratory studies and urinalysis are ordered and currently pending. Which of the following is the pathophysiology of this patient's chief complaint? A) Decreased lower esophageal tone B) Esophageal fibrosis C) Increased lower esophageal tone D) Spastic cricopharyngeal muscle E) Uncoordinated esophageal peristalsis
B, Scleroderma This patient has clinical features suggestive of scleroderma given her taut skin (young-appearing skin), hand pain in the cold (Raynaud syndrome), esophageal reflux (GERD from fibrosis of the esophagus), kidney disease (suggested by her hypertension), and interstitial lung disease (as seen on CT with the diffuse fibrosis). Scleroderma is a systemic collagen vascular disease with inappropriate accumulation of collagen in various organ systems. Organ systems that can be affected include renal (presenting with hypertension, hyperkalemia, and an elevated BUN/creatinine), pulmonary (presenting with pulmonary fibrosis and dyspnea), dermatologic (presenting with taut and smooth skin as well as Raynaud disease), and GI symptoms (in particular, GERD secondary to collagen deposition in the esophagus). There is no definitive treatment of scleroderma and management involves treating each of the underlying organ systems affected. The treatment of GERD in scleroderma typically involves administration of PPIs and small meals that are low in fat. Figure A shows diffuse fibrosis of the lungs concerning for interstitial lung disease which is commonly seen in scleroderma. Incorrect Answers: Answer 1: Decreased lower esophageal tone is the pathophysiology of GERD which presents with burning retrosternal pain typically after a meal. Though it could be the cause of this patient's symptoms, her other findings suggestive of scleroderma make fibrosis a more likely etiology. Answer 3: Increased lower esophageal tone is the pathophysiology of achalasia which presents in young men with regurgitation of undigested food particles. Barium swallow would show a bird beak appearance and manometry would demonstrate increased lower esophageal tone. Treatment involves a myotomy. Answer 4: Spastic cricopharyngeal muscle describes a Zenker diverticulum which presents with regurgitation of undigested food in an elderly patient and a sensation that food is stuck in the throat. Barium swallow will show an outpouching in the proximal esophagus and treatment involves surgical resection. Answer 5: Uncoordinated esophageal peristalsis describes diffuse esophageal spasm which presents with episodic chest pain and dysphagia secondary to intermittent esophageal spasm. Treatment involves administration of calcium channel blockers. Bullet Summary: GERD is a common complication of scleroderma and occurs secondary to collagen deposition in the esophagus.
A 50-year-old male visits his primary care physician with skin lesions on his knees and elbows. He reports joint pain, and physical examination reveals severe swelling of the fingers on both hands. Tests for serum rheumatoid factor are negative. Which of the following pairs of adjectives most likely characterize the patient's skin lesions: A) Honey-colored, crusting B) Irregular, depigmented C) Silver, scaly D) Velvety, hyperpigmented E) Non-blanching, hemorrhagic
C, Psoriatic Arthritis The patient's presentation is most consistent with psoriasis complicated by psoriatic arthritis. Psoriatic lesions present as dark red plaques or papules with silvery-white scales and sharp margins. Psoriasis lesions most commonly appear on the elbows and knees. Less than one third of patients with psoriasis will develop psoriatic arthritis, marked by joint pain, stiffness, and joint abnormalities including finger inflammation (dactylitis) and deformity. Nail pitting is common. Patients with psoriatic arthritis are seronegative for rheumatoid factor, making the disease a seronegative spondyloarthropathy. Luba and Stulberg review chronic plaque psoriasis. Psoriasis is a T-cell mediated autoimmune disease. Management includes topical steroids and vitamin D derivatives, including calcipotriene. Patients who don't respond to topical treatment may be candidates for systemic treatment with immuno-modulatory drugs such as alefacept, efalizumab, and etanercept. Yamamoto reviews psoriatic arthritis from a dermatological perspective, noting that it is important for timely diagnosis as joint damage occurs early on in the disease. In the majority of patients, cutaneous manifestations of psoriasis precede joint involvement, as such, dermatologists should evaluate psoriasis patients for the presence of joint involvment. Dactylitis, enlarged sausage-like fingers, is one of the cardinal features of psoriatic arthritis, and highly specific for the diagnosis. Image A depicts typical hands of a patient with psoriatic arthritis. Illustration A shows a typical psoriatic lesion. Incorrect answers: Answer 1: Honey-colored, crusting lesions are found in impetigo, a superficial infection of the skin. Answer 2: Irregular, depigmented lesions classically characterize vitiligo. Answer 4: Velvety, hyperpigmented skin characterizes acanthosis nigricans. The condition usually localizes to body folds and is associated with insulin resistance, obesity, polycystic ovarian syndrome, paraneoplastic syndromes, and others. Answer 5: Non-blanching, hemorrhagic lesions describe ecchymoses, purpura, or petechiae. The three lesions are differentiated by size, with ecchymoses characterizing lesions larger than 1 cm, purpura characterizing lesions measuring 5 mm to 1 cm, and petachiae characterizing lesions less than 5 mm.
A 32-year-old woman presents with a three-month history of difficulty swallowing. She says that it occurs with both solids and liquids with the sensation that food is getting stuck in her throat. Additionally, the patient reports that while shoveling snow this past winter, she had noticed that her hands would lose their color and become numb. She denies any cough, regurgitation, joint pains, shortness of breath, fever, or changes in weight. She does not smoke or drink alcohol. The patient's physical exam is within normal limits, although she does appear to have thickened, tight skin on her fingers. She does not have any other skin findings. Which antibody will most likely be found on serological study in this patient? A) Anti-mitochondrial antibodies B) Anti-centromere antibodies C) Anti-topoisomerase antibodies D) Anti-U1-RNP antibodies E) Anti-CCP antibodies
B, Scleroderma This patient presents with esophageal dysmotility, sclerodactyly, and Raynaud's phenomenon without any other systemic symptoms, which is most consistent with limited scleroderma, otherwise known as CREST syndrome. CREST syndrome is associated with anti-centromere antibodies and anti-nuclear antibodies. CREST syndrome (also known as cutaneous systemic sclerosis) is the limited form of scleroderma, and includes Calcinosis (deposits of calcium nodules in the dermis), Raynaud's phenomenon, Esophageal Dysmotility, Sclerodactyly (sclerosis can also affect face), and Telangiectasias. The most specific antibody for CREST syndrome are anti-centromere antibodies. Incorrect Answers: Answer 1: Anti-mitochondrial antibodies are associated with primary biliary cirrhosis. Answer 3: Anti-topoisomerase antibodies, also known as anti-Scl70, are associated with systemic sclerosis (diffuse scleroderma). Anti-centromere antibodies are much more common in the limited scleroderma than in the diffuse form, and anti-topoisomerase antibodies are more common in the systemic form than in the limited form. Systemic sclerosis is also associated with anti-nuclear antibodies. Answer 4: Anti-U1-RNP antibodies are found in mixed connective tissue disorder (MCTD). MCTD can have many symptoms consistent with scleroderma; however, this diseaes would also present with other connective tissue symptoms including myalgias, arthralgias, and rashes. Answer 5: Anti-CCP (anti-cyclic citrullinated peptide) antibodies are specific for rheumatoid arthritis. Bullet Summary: Limited scleroderma (or CREST syndrome) is associated with anti-centromere antibodies and anti-nuclear antibodies.
A 46-year-old woman presents to the emergency department with progressive dyspnea and chest pain. She reports that her symptoms started 1 week ago and have gotten progressively worse. The chest pain is left-sided and is exacerbated by coughing or deep breaths. She also endorses a 6-month history of joint pains involving her knees, elbows, and digits. She does not have a significant medical or surgical history. She takes ibuprofen as needed. She works as a pre-school teacher. The patient's temperature is 99°F (37.2°C), blood pressure is 120/78 mmHg, pulse is 89/min, and respirations are 17/min with an oxygen saturation of 93% on room air. On physical examination, a friction rub upon inspiration/expiration and crackles are appreciated at the base of the left lung. She has an erythematous rash that spans the bilateral cheeks and nose. There are also scattered ecchymoses on her arms and legs. A chest radiograph shows a small left-sided pleural effusion. A complete blood count is obtained, as shown below: Hemoglobin: 9 g/dL Hematocrit: 28% Leukocyte count: 1,500/mm^3 with normal differential Platelet count: 80,000/mm^3 A urinalysis shows elevated protein levels. Serologic antibodies are pending. Which of the following is the primary cause of the patient's lab results? A) Hematologic malignancy B) Immune-mediated destruction C) Mechanical shearing D) Splenic sequestration E) Viral infection
B, Systemic Lupus Erythematous The patient is presenting with pleuritis (i.e., pleuritic chest pain and friction rub), pleural effusion, arthralgias, a malar rash, proteinuria, and pancytopenia, which are consistent with systemic lupus erythematous (SLE). Pancytopenia that occurs with SLE is primarily due to immune-mediated destruction. Systemic lupus erythematous (SLE) is a systemic autoimmune disease that presents with characteristic multi-organ symptoms. The acronym SOAP BRAIN MD is helpful for remembering the associated findings: Serositis (i.e., pleuritis, pericarditis), Oral ulcers, Arthritis, Photosensitivity, Blood (i.e., pancytopenia), Renal (i.e., proteinuria), ANA, Immunologic (i.e., anti-dsDNA), Neurologic (i.e., psychiatric disorders, seizures, and headaches), Malar rash, and Discoid rash. Pancytopenia (i.e., anemia, leukopenia, and thrombocytopenia) is caused by immune-mediated destruction. Antibodies associated with SLE cause immune-mediated bone marrow failure and peripheral cell destruction. Incorrect Answers: Answer 1: Hematologic malignancy, such as leukemia or lymphoma, will usually present with leukocytosis not pancytopenia. Answer 2: Mechanical shearing (i.e., mechanical heart valves) can be a cause of hemolytic anemia but would not explain the leukopenia or thrombocytopenia. Answer 4: Splenic sequestration may play a role in pancytopenia associated with SLE; however, the primary etiology is immune-mediated destruction. Splenic sequestration is associated with sickle cell disease and can cause splenomegaly. Answer 5: Viral infection, such as with parvovirus, can be a cause of aplastic anemia, resulting in pancytopenia. This is especially true for patients with sickle cell disease. SLE better explains the patient's multi-organ presentation including proteinuria and a malar rash. Bullet Summary: Systemic lupus erythematosus can cause pancytopenia via immune-mediated destruction.
A 27-year-old woman presents to her primary care physician with a chief complaint of pain in her hands, shoulders, and knees. She states that the pain has lasted for several months but seems to have worsened recently. Any activity such as opening jars, walking, or brushing her teeth is painful. The patient has a past medical history of a suicide attempt in college, constipation, anxiety, depression, and a sunburn associated with surfing which was treated with aloe vera gel. Her temperature is 99.5°F (37.5°C), blood pressure is 137/78 mmHg, pulse is 92/min, respirations are 14/min, and oxygen saturation is 98% on room air. Laboratory values are obtained and shown below. Hemoglobin: 9 g/dL Hematocrit: 33% Leukocyte count: 2,500/mm^3 with normal differential Platelet count: 107,000/mm^3 Serum: Na+: 139 mEq/L Cl-: 102 mEq/L K+: 4.4 mEq/L HCO3-: 24 mEq/L BUN: 21 mg/dL Glucose: 90 mg/dL Creatinine: 1.0 mg/dL Ca2+: 10.2 mg/dL AST: 12 U/L ALT: 10 U/L Which of the following is the most likely to be found in this patient? A) Anti-cyclic citrullinated peptide antibodies B) Anti-dsDNA antibodies C) Anti-histone antibodies D) Degenerated cartilage in weight bearing joints E) IgM against parvovirus B19
B, Systemic Lupus Erythematous This patient is presenting with arthralgias, pancytopenia, possible malar rash, and a history of depression/abnormal behavior. Epidemiologically, the most likely diagnosis is systemic lupus erythematosus (SLE) which is associated with anti-dsDNA antibodies. SLE presents with at least 4 criteria from SOAP BRAIN MD - Serositis, Oropharyngial ulcers, Anti-nuclear antibodies, Photosensitivity, Blood disorders (pancytopenia and hemolytic anemia), Renal abnormalities (proteinuria), Arthralgias, Immune disease (hemolytic anemia and anti-dsDNA), Neurologic (psychosis, stroke, and seizure), Malar rash, and Discoid rash. The most common presenting symptoms for SLE can often be just arthralgias/arthritis with vague other symptoms suggestive of a rheumatologic disorder. Anti-nuclear antibodies are very sensitive for this pathology and anti-dsDNA antibodies are very specific. Incorrect Answers: Answer 1: Anti-cyclic citrullinated peptide is associated with rheumatoid arthritis which is a possible diagnosis in this patient, but it is less likely epidemiologically given her age and other symptoms of pancytopenia and a history of sunburn which is likely a malar rash or photosensitivity. Answer 3: Anti-histone antibodies would be found in drug-induced lupus. This patient's medication history is not known; however, this is a less likely diagnosis compared to SLE. Answer 4: Degenerated cartilage in weight bearing joints describes osteoarthritis which tends to affect the knees and hips in older patients. Answer 5: IgM against parvovirus B19 describes a parvovirus B19 infection which commonly presents with arthralgias; however, her other symptoms point toward a diagnosis of SLE. Bullet Summary: Arthritis/arthralgias are often the most common presenting symptom for SLE.
A 33-year-old woman presents to her primary care physician with bilateral joint pain. She says that the pain has been slowly worsening over the past 3 days. Otherwise, she complains of fatigue, a subjective fever, and a sunburn on her face which she attributes to gardening. The patient is an immigrant from Spain and works as an office assistant. She is not aware of any chronic medical conditions and takes a multivitamin daily. Her temperature is 98.7°F (37.1°C), blood pressure is 125/64 mmHg, pulse is 80/min, respirations are 13/min, and oxygen saturation is 98% on room air. Physical exam reveals bilateral redness over the maxillary prominences. Which of the following is most likely to be seen in this patient? A) Decreased anti-dsDNA antibodies B) Decreased complement levels C) Increased anti-centromere antibodies D) Increased anti-cyclic citrullinated peptide antibodies E) Increased anti-topoisomerase antibodies
B, Systemic Lupus Erythematous This patient is presenting with bilateral arthralgias, a malar rash, and systemic symptoms, which are associated with a diagnosis of systemic lupus erythematosus (SLE). SLE is associated with decreased complement levels. SLE presents with SOAP BRAIN MD: Serositis Oropharyngeal ulcers Arthritis Photosensitivity Blood disorders (autoimmune hemolytic anemia) Renal abnormalities (elevated creatinine/BUN) Anti-nuclear antibodies Immune disease (anti-dsDNA antibodies) Neurological symptoms Malar rash Discoid rash The most common presentation during a flare is arthritis. During an acute flare, complement levels will be decreased (secondary to constitutive activation), and anti-dsDNA levels will be elevated. Incorrect Answers: Answer 1: Decreased anti-dsDNA antibodies is incorrect because anti-dsDNA antibodies will be elevated in an acute SLE flare. Answer 3: Increased anti-centromere antibodies would be seen in CREST syndrome which presents with Calcinosis cutis, Raynaud phenomenon, Esophageal dysmotility, Sclerodactyly, and Telangiectasias. Answer 4: Increased anti-cyclic citrullinated peptide antibodies would be seen in rheumatoid arthritis which would present with bilateral joint pain worse in the morning which improves with activity as well as systemic symptoms such as malaise/fatigue. The malar rash described in the case points more toward SLE. Answer 5: Increased anti-topoisomerase antibodies would be seen in scleroderma which presents with sclerodactyly, smooth skin, renal failure/abnormalities, and esophageal dysmotility. Bullet Summary: In an acute lupus flare, complement levels will be decreased, and anti-dsDNA antibodies will be elevated.
A 29-year-old woman presents to the emergency department with joint pain and a notable rash. She has had joint pain for the past 12 months but noticed the rash recently as well as generalized malaise. She states her joint pain is symmetric, in her upper extremities, and is worse in the morning. Her temperature is 97.6°F (36.4°C), blood pressure is 111/74 mmHg, pulse is 83/min, respirations are 14/min, and oxygen saturation is 98% on room air. Laboratory studies are ordered as seen below. Hemoglobin: 10 g/dL Hematocrit: 30% Leukocyte count: 6,800/mm^3 with normal differential Platelet count: 207,000/mm^3 Serum: Na+: 140 mEq/L Cl-: 101 mEq/L K+: 4.9 mEq/L HCO3-: 21 mEq/L BUN: 30 mg/dL Glucose: 120 mg/dL Creatinine: 1.8 mg/dL The patient is ultimately admitted to the hospital. Which of the following is the most appropriate test to monitor her disease progression? A) Anti-CCP B) Anti-dsDNA C) Anti-nuclear antibody D) Anti-topoisomerase E) Rheumatoid factor
B, Systemic Lupus Erythematous This patient is presenting with joint pain, a malar rash, and kidney disease suggesting a diagnosis of systemic lupus erythematosus (SLE). In patients with SLE, the level of anti-dsDNA antibody can be used to monitor disease activity. SLE is characterized by the production of auto-antibodies that, along with immune complex formation, cause the multi-organ manifestations of the disease. One such auto-antibody is anti-dsDNA. Anti-dsDNA has high specificity for SLE and is rarely found in the presence of other autoimmune conditions. There is an especially strong correlation between IgG anti-dsDNA and glomerulonephritis in the context of SLE. Serum C3 and C4 levels are also used to track SLE disease activity. The diagnosis of SLE should be suspected when a patient presents with symptoms including serositis, oropharyngeal ulcers, anti-nuclear antibodies (ANA), photosensitivity (including a malar or discoid rash), hemolytic anemia, kidney disease among many other possible presenting symptoms. Figure A is the butterfly/malar rash seen in SLE. Incorrect Answers: Answers 1 & 5: Anti-CCP is a more specific antibody found in rheumatoid arthritis when compared to rheumatoid factor. Rheumatoid arthritis would present with symmetric arthritis which is worse in the morning and improves throughout the day. Other findings could include malaise and anemia of chronic disease. This patient's arthralgias in the setting of kidney disease and a malar rash suggest a diagnosis of SLE. Answer 3: Anti-nuclear antibody (ANA) is useful in the initial diagnosis of SLE but is not used to track disease progress. It may be the most appropriate initial step in management when considering a diagnosis of SLE but does not monitor disease progress. Answer 4: Anti-topoisomerase could be elevated in scleroderma which presents with taut skin, sclerodactyly, interstitial lung disease, GERD, small bowel bacterial overgrowth, and kidney disease. Bullet Summary: Anti-dsDNA can be used to monitor flares in SLE.
A 34-year-old man presents to the clinic complaining of joint stiffness for the past 2 weeks. He reports that his bilateral distal interphalangeal joints are especially stiff when he wakes up but improve after 30-40 minutes. He denies any fever, trauma, weight loss, or gastrointestinal symptoms, but endorses an oral ulcer that he attributes to recent stress. His medical history is significant for a ventricular arrhythmia that he started taking procainamide for. A physical examination demonstrates skin findings shown in Figure A and mild swelling of the right index finger. Which of the following is the most specific finding you would expect in this patient? A) Anti-dsDNA antibodies B) Anti-histone antibodies C) HLA-B27 positivity D) Increased erythrocyte sedimentation rate (ESR) E) Rheumatoid factor
C, Psoriatic Arthritis This patient likely has psoriatic arthritis as demonstrated by his symmetrical peripheral joint stiffness, dactylitis (swollen right index finger), and psoriatic skin lesions. Psoriatic arthritis is strongly associated with HLA-B27 positivity. Psoriatic arthritis describes the seronegative spondyloarthritis that is associated with the autoimmune disease psoriasis. Patients with the disease classically present with asymmetric and patchy joint pain and stiffness. However, some patients may present with symmetrical joint involvement that may be indistinguishable from rheumatoid arthritis. Other symptoms include dactylitis ("sausage fingers"), uveitis, inflammatory back pain, and enthesitis. Skin findings include psoriatic lesions that are characteristically red, itchy plaques with silvery scales. Psoriatic arthritis, along with other seronegative spondyloarthropathies (e.g., ankylosing spondylitis, inflammatory bowel disease, and reactive arthritis), is strongly associated with HLA-B27 positivity. Figure A is a clinical image demonstrating the red lesions with silvery scale characteristic of psoriasis. Incorrect Answers: Answer 1: Anti-dsDNA antibodies are associated with patients with systemic lupus erythematosus (SLE). The classic presentation includes malar rash, joint pain, and fever and is more common in a female of reproductive age. This patient's clinical presentation is more in line with psoriatic arthritis. Answer 2: Anti-histone antibodies are found in patients with drug-induced lupus, often after medications such as hydralazine and/or procainamide. Although this patient is taking procainamide, his symptoms (e.g., psoriatic skin lesions and joint stiffness) are not in line with that of drug-induced lupus. Answer 4: Increased erythrocyte sedimentation rate is a nonspecific marker of inflammation that is often found in patients with autoimmune diseases such as SLE or rheumatoid arthritis. Answer 5: Rheumatoid factor is associated with rheumatoid arthritis, which can present similarly to psoriatic arthritis. However, the presence of psoriatic skin lesions on this patient makes psoriatic arthritis more likely. Bullet Summary: Psoriatic arthritis, along with other seronegative spondyloarthropathies, is associated with HLA-B27 positivity.`
A 25-year-old woman comes to the physician's office with complaints of joint pain. She has had multiple joints ache over the past few months and feels like it is getting worse. She has no past medical history. She has had unprotected sex with 3 male partners. Her BMI is 37 kg/m^2. On exam she also has a faint skin rash on her face which she says gets worse when she is outside. Further workup for this reveals a urinalysis shown in Figure A. The patient reveals that in the past she has had issues with "leg swelling". She has been frequently told that she has tested positive for syphilis even though a second test is usually negative. Her coagulation studies are as follows: Bleeding time - Normal Activated partial thromboplastin time - Prolonged Prothrombin time - Normal Platelets - 205,000/uL The patient is at greatest risk for which of the following complications? A) Sclerodactyly B) Menorrhagia C) Spontaneous abortion D) Megaloblastic anemia E) Chancre formation
C, Systemic Lupus Erythematosus The patient in the question likely has systemic lupus erythematosis (SLE) complicated by antiphospholipid syndrome. This places her at higher risk for spontaneous abortion. Antiphospholipid syndrome is an autoimmune disease that typically coexists with lupus. Glomerulonephritis, a photosensitive rash (likely a malar rash), and arthralgias are suggestive of lupus. Antiphospholipid syndrome is caused by circulating antiphospholipid antibodies that place the patient in a hypercoagulable state. As a result, patients are more predisposed to spontaneous abortions in the first trimester and thromboembolic disease. It is diagnosed by the presence of anticardiolipid antibodies and anti-B2-glycoprotein antibodies. The antibodies can cause a false positive VDRL, so confirmatory testing for syphilis is likely negative. Patients may also show an elevated PTT because the antibodies interfere with the test itself (the patient is not in currently anticoagulated). Figure A shows dysmorphic RBCs which is a sign of glomerulonephritis, the most common form of kidney disease seen in SLE. Incorrect Answers: Answer 1: Sclerodactyly is typically seen with scleroderma. It is not typically seen in patients with lupus. Answer 2: Menorrhagia would be seen in patients who are hypocoagulable or in a pro-bleeding state. The elevated PTT in this patient is a result of the antibodies' interference with the clotting test. Answer 4: Megaloblastic anemia is typically seen with B12 or folate deficiency. This patient does not have any risk factors for this. Answer 5: Chancre formation would be seen with syphilis. Though the patient has tested positive for syphilis (likely false positive VDRL due to the antiphospholipid antibodies), confirmatory testing for her has been negative. Bullet Summary: Antiphospholipid syndrome is an autoimmune disorder most commonly found with lupus that causes a hypercoagulable state leading to thrombosis and spontaneous abortions, with lab findings of lupus anticoagulant, anticardiolipin, and anti-B2 glycoprotein antibodies and a prolonged PTT.
A 50-year-old woman presents to her primary care physician for muscle weakness. Her symptoms have progressively worsened over the course of several months, where she has experienced difficulty getting up from a chair, climbing the stairs, and placing her groceries on the top shelves of her kitchen. She also reports feeling tired lately and is unsure when this began. Physical examination is notable for muscle weakness in the deltoids and hip flexors with 2+ biceps, triceps, patellar, and ankle tendon reflexes. Laboratory testing demonstrates a creatine kinase level of 4,200 U/L (normal is 10-70 U/L), lactate dehydrogenase level of 150 U/L (normal is 45-90 U/L), and thyroid-stimulating hormone level of 1.5 µU/mL (normal is 0.5-5.0 µU/mL). Which of the following is the best initial treatment for this patient? A) Azathioprine B) Intravenous immunoglobulin C) Physostigmine D) Prednisone E) Thyroxine
D, Dermatomyositis / Polymyositis This patient's proximal muscle weakness, elevated muscle enzymes, and absent dermatologic findings are highly concerning for polymyositis. Initial treatment is with prednisone, a glucocorticoid. Polymyositis is an idiopathic inflammatory myopathy characterized by proximal skeletal muscle weakness and elevated markers of muscle inflammation (e.g., creatine kinase, lactate dehydrogenase, and aldolase). Initial therapy is with systemic glucocorticoids, such as prednisone. Glucocorticoid-sparing agents are typically added with initial therapy to decrease the amount of glucocorticoid given. Early in the treatment course, physical therapy and rehabilitation should be part of a patient's treatment regimen. Incorrect Answers: Answer 1: Azathioprine is a first-line steroid-sparing agent that is added to the glucocorticoid when treating polymyositis. It can be used in monotherapy in cases where glucocorticoids are contraindicated, which is not the case here. Answer 2: Intravenous immunoglobin is a treatment option for patients with Guillain-Barre syndrome. This would be the case if the patient had a respiratory or gastrointestinal infection a few weeks prior to the development of weakness, along with reduced or absent deep tendon reflexes. Answer 3: Physostigmine is a treatment option for patients with myasthenia gravis, which presents with fatigable muscle weakness. Patients typically present with ocular findings such as ptosis and diplopia. Answer 5: Thyroxine is the treatment for hypothyroid myopathy. This is not likely in this patient given her normal thyroid-stimulating hormone level, a sensitive test for hypothyroidism. Bullet Summary: Initial treatment for polymyositis and dermatomyositis is with systemic glucocorticoids, such as prednisone.
A 39-year-old woman comes into your office with multiple complaints of recent onset. She has been feeling fatigued and has started having leg and thigh pain bilaterally, along with weakness. In addition, she states that she recently started noticing some redness around her eyes along with a rash on her shoulders as shown in Figure A. Of note, she takes simvastatin for her hyperlipidemia, and took a short course of steroids recently for an allergic reaction. Her vitals are within normal limits. Her CK is 400 U/L, ESR is 70 mm/hr, and ferritin is 500 mcg/L. What is the most likely diagnosis? A) Inclusion body myositis B) Corticosteroid induced myopathy C) Statin induced myopathy D) Dermatomyositis E) Polymyositis
D, Dermatomyositis / Polymyositis A 39-year-old woman with a rash around her eyes (heliotrope rash), along with a rash on her shoulders (shawl sign), elevated CK, ESR, and ferritin, in the setting of proximal muscle weakness is most likely suffering from dermatomyositis. Dermatomyositis is an inflammatory myopathy with characteristic skin manifestations involving the eyes, the upper neck and shoulders, and the dorsum of hands. It has an association with other connective tissue disorders along with a higher risk of future and concurrent malignancy. Patient management includes long-term physical therapy along with antihistamines, sunscreen for the rashes, and oral corticosteroids for musculoskeletal pain. The goal of therapy is typically to maintain function without significant side effects. Koler et al. discuss the diagnosis and treatment of dermatomyositis. They state that the diagnosis of dermatomyositis typically involves skin lesions (discussed above), along with proximal muscle weakness, elevated levels of CK, ferritin (acute phase reactant), and ESR, muscle pain, and electromyography showing muscle changes. The note that anti-Jo-1 antibodies may also be positive in these patients. Lastly, they recommend that in patients with disease resistant to corticosteroids, immunotherapies such as methotrexate and cyclophosphamide may be used. The Muscle Study Group discusses a randomized controlled trial involving etanercept, a TNF-alpha inhibitor, as a mode of therapy for dermatomyositis. In their trial, they note that 5 of 11 subjects in the etanercept arm were successfully weaned off prednisone, while 0 of the 5 subjects in the placebo arm were able to be weaned off. They also state that the median of the average prednisone dosage after week 24 was 29.2 mg/day in the placebo group and 1.2 mg/day in the etanercept group (p = 0.02). Figure A shows a symmetric "cloak-like" rash involving shoulders, neck, and possibly the chest wall. It is denoted as the "shawl sign" and is common in dermatomyositis. Incorrect Answers: Answer 1: Inclusion body myositis typically affects distal forearm and arm muscles and may cause bulbar symptoms of dysphagia. Answers 2 and 3: The patient may have corticosteroid or statin induced myopathy, but her heliotrope rash (around the eyes) and the shawl sign are more typical of an inflammatory myositis. Answer 5: Polymyositis also commonly affects proximal muscles and results in weakness, but it more commonly affects the lungs and the GI tract. It also does not present with a rash.
A 50-year-old man arrives to the clinic complaining of progressive weakness. He explains that for 3 months he has had difficulty climbing the stairs, which has now progressed to difficulty getting out of a chair. He denies diplopia, dysphagia, dyspnea, muscle aches, or joint pains. He denies weight loss, weight gain, change in appetite, or heat or cold intolerance. He reports intermittent low-grade fevers. He has a medical history significant for hypertension and hyperlipidemia. He has taken simvastatin and losartan daily for the past 6 years. His temperature is 99.0°F (37.2°C), blood pressure is 135/82 mmHg, and pulse is 76/min. Cardiopulmonary examination is normal. The abdomen is soft, non-tender, non-distended, and without hepatosplenomegaly. Muscle strength is 3/5 in the hip flexors and 4/5 in the deltoids, biceps, triceps, patellar, and Achilles tendon reflexes are 2+ and symmetric. Sensation to pain, light touch, and vibration are intact. Gait is cautious, but grossly normal. There is mild muscle tenderness of his thighs and upper extremities. There is no joint swelling or erythema and no skin rashes. A complete metabolic panel is within normal limits. Additional lab work is obtained as shown below: Serum: Na+: 141 mEq/L Cl-: 100 mEq/L K+: 4.3 mEq/L HCO3-: 23 mEq/L Urea nitrogen: 18 mg/dL Glucose: 128 mg/dL Creatinine: 1.0 mg/dL Alkaline phosphatase: 69 U/L Aspartate aminotransferase (AST): 302 U/L Alanine aminotransferase (ALT): 210 U/L TSH: 6.9 uU/mL Thyroxine (T4): 5.8 ug/dL Creatine kinase: 4300 U/L C-reactive protein: 11.9 mg/L Erythrocyte sedimentation rate: 37 mm/h Which of the following is the most accurate diagnostic test? A) Autoantibodies B) Electromyography C) Fine needle aspiration D) Muscle biopsy E) Statin cessation
D, Dermatomyositis / Polymyositis The patient is presenting with progressive, painless proximal muscle weakness, and elevated creatine kinase, transaminases, and inflammatory markers, which is suspicious for polymyositis. Polymyositis is an inflammatory myopathy which presents at age 40-50 with proximal muscle weakness (i.e., difficulty climbing stairs, getting out of a chair, lifting heavy objects, and combing hair). Joint pain or swelling may be present as well as muscle tenderness. The condition may be associated with interstitial lung disease, myocarditis, and malignancy. Labs will show elevated muscle enzymes (i.e., creatine kinase and transaminases) and inflammatory markers (i.e., C-reactive protein), and some patients may have the presence of autoantibodies (i.e., ANA and anti-Jo-1). Diagnosis is confirmed by muscle biopsy demonstrating endomysial mononuclear patchy necrosis. Treatment is corticosteroids. Incorrect Answers: Answer 1: Autoantibodies may be present in polymyositis but are not specific. Rather, creatine kinase and aldolase are the best initial tests while a muscle biopsy is the most accurate diagnostic test. Answer 2: Electromyography may be used to show widespread denervation and fibrillation characteristic of amyotrophic lateral sclerosis (ALS). ALS presents with both lower motor neuron signs (i.e., atrophy, fasciculations, hyporeflexia, and weakness) and upper motor neuron signs (i.e., spasticity, hyperreflexia, and stiffness). Answer 3: Fine needle aspiration may be useful for the diagnosis of a thyroid nodule. This patient has an elevated TSH but a normal T4, which is suspicious for subclinical hypothyroidism. Hypothyroid myopathy can cause muscle weakness and an elevated CK, but it would also present with myalgias, delayed reflexes, and other symptoms of hypothyroidism. Answer 5: Statin cessation is important for the management of statin-induced myopathy. Statin myopathy typically presents with mild muscular pain and elevated creatine kinase levels soon after starting statin therapy. This patient has been on chronic therapy. Bullet Summary: Polymyositis presents with symmetrical pain and proximal muscle weakness and is most accurately diagnosed with a muscle biopsy.
A 57-year-old woman presents to her primary care physician with a chief complaint of weakness. She has felt weak for the past 6 months but states that her symptoms have worsened recently. The patient also reports feeling fatigued, depressed, and has gained 10 pounds which she attributes to feeling too weak to go to the gym. She was recently treated for an ear infection with an antibiotic which she completed 3 weeks ago, and experienced a rash which she believes was an allergic reaction to her treatment. The patient has a past medical history of diabetes mellitus that is well-controlled with insulin and metformin. She has been admitted multiple times for hypoglycemia secondary to missing meals. Her temperature is 99.5°F (37.5°C), blood pressure is 127/68 mmHg, pulse is 80/min, respirations are 17/min, and oxygen saturation is 98% on room air. Physical exam is notable for 2/5 strength in the patient's upper and lower extremities, sparse fine hair on her body, and a rash on her face and arms. Laboratory values are ordered as seen below. Serum: Na+: 139 mEq/L K+: 3.3 mEq/L Cl-: 100 mEq/L HCO3-: 25 mEq/L BUN: 20 mg/dL Glucose: 70 mg/dL Creatinine: 1.1 mg/dL Ca2+: 10.2 mg/dL AST: 12 U/L ALT: 10 U/L A muscle biopsy is obtained as seen in Figure A. Which of the following is the most likely explanation for this patient's presentation? A) Dermatomyositis B) Hypokalemia C) Hypothyroidism D) Inclusion body myositis E) Polymyositis
D, Dermatomyositis / Polymyositis This patient is presenting with a biopsy suggestive of inclusion body myositis. Inclusion body myositis presents with an insidious onset of weakness, a mildly elevated CRP and CK, and a muscle biopsy demonstrating endomysial inflammation with basophilic-rimmed vacuoles within the muscle fiber sarcoplasm. Electromyography is typically abnormal, and the most accurate/confirmatory test is a muscle biopsy (Figure A). Figure A is a muscle biopsy demonstrating endomysial inflammation with basophilic-rimmed vacuoles within the muscle fiber sarcoplasm. Incorrect Answers: Answers 1 & 5: Dermatomyositis and polymyositis present similarly with muscle weakness and an elevated CK, CRP, and aldolase. However, dermatomyositis presents with dermatologic findings. A biopsy would demonstrate inflammation surrounding the muscle fascicle (dermatomyositis) or within the muscle fascicle (polymyositis). Answer 2: Hypokalemia could present with weakness and cardiac abnormalities. Though this patient is hypokalemic, in the setting of a biopsy demonstrating inclusion body myositis a better explanation for her symptoms is inclusion body myositis. Answer 3: Hypothyroidism presents with depression, fatigue, scarce lanugo, and weight gain (as this patient has); however, her symptoms of weakness have a confirmed cause via the results on biopsy. Bullet Summary: Inclusion body myositis presents with weakness and an elevated CRP/CK with a muscle biopsy demonstrating endomysial inflammation with basophilic-rimmed vacuoles within the muscle fiber sarcoplasm.
A 41-year-old woman comes to the clinic complaining of progressive shortness of breath and fatigue for the past 2 years. She reports that "it gets difficult to breathe, especially when I try to exercise at the gym." According to the patient, she first noticed the symptoms several years ago, but it has worsened to the point that 2 minutes of cardiovascular exercise will leave her dyspneic. Past medical history includes hypertension and 1 miscarriage. She reports smoking 1 pack per day for the past 20 years. Physical examination is significant for a loud S2, jugular venous pulse (JVP) of 10cm H2O, and bilateral lower extremity edema. The patient's skin findings are shown in Figure A. What systemic disease best explains this patient's presenting condition? A) Dermatomyositis B) Lambert-Eaton syndrome C) Polymyalgia rhuematica D) Scleroderma E) Systemic lupus erythematous
D, Scleroderma This patient's symptoms (e.g., progressive dyspnea, prominent S2, lower extremity edema, and jugular venous distension with clear lung sounds) suggest pulmonary hypertension. Patients with scleroderma (as seen with this patient's taut and swollen skin in Figure A) are predisposed to getting pulmonary hypertension. Scleroderma comprises a variety of conditions that involve autoimmunity, non-inflammatory vasculopathy, and collagen deposition with fibrosis. Patients commonly present with sclerosis of the skin, which manifests as edematous, taut skin without wrinkles. Diffuse scleroderma is marked by widespread skin involvement, rapid progression, and early visceral involvement (sclerosis of renal, pulmonary, cardiovascular, and gastrointestinal systems). Of note, pulmonary hypertension is the most common cause of death in patients with scleroderma. Figure A demonstrates a clinical picture of a hand with taut, swollen skin without wrinkles. Incorrect Answers: Answer 1: Dermatomyositis is an autoimmune condition characterized by progressive symmetrical proximal muscle weakness and skin involvement (e.g., malar rash, heliotrope rash, and "shawl and face" rash). It is associated with occult malignancies. Answer 2: Lambert-Eaton syndrome is a paraneoplastic syndrome often associated with small cell lung cancer. Patients present with proximal muscle weakness. Despite this patient's extensive smoking history, her physical exam findings are more suggestive of scleroderma. Answer 3: Polymyalgia rhuematica presents with pain and stiffness in shoulders and hips, weight loss, malaise, and fever. It often presents in patients > 50 years of age and is associated with giant cell arteritis. Answer 5: Systemic lupus erythematous is an autoimmune condition characterized by rash, joint pain, and fever. This patient's symptoms do not corroborate with lupus. Bullet Summary: Patients with scleroderma, which presents with taut, swollen skin without wrinkles, are predisposed to developing pulmonary hypertension.
An 8-year-old boy is brought to the emergency room for a possible elbow dislocation. The parents report that he was playing in the playground when he tripped and fell on his outstretched right arm. Shortly following the fall, the patient began to cry in pain and kept his right elbow in a flexed position. Any movement or attempt to touch the arm would result in pain. The patient is relatively healthy and has been growing normally with regards to his growth curve. His past medical history consists of 2 ankle sprains and 1 left shoulder dislocation. A physical examination demonstrates a boy in distress who maintains good eye contact and appropriate responses. There is a very prominent right olecranon and a forearm that appears foreshortened. The sensation is intact though strength was limited due to pain. Radial pulses are present bilaterally. Skin finds are shown in Figure A. What is the most likely explanation for this patient's findings? A) Child abuse B) Decreased production of type I collagen C) Defective ATP7A leading to impaired copper absorption D) Mutation in type V collagen E) Mutation of the FBN1 gene
D, Ehlers-Danlos Syndrome This patient likely has Ehlers-Danlos syndrome as demonstrated by his history of recurrent dislocations/sprains and hyperelastic skin. The classical type of Ehlers-Danlos is due to a mutation in type V collagen. Ehlers-Danlos syndrome describes a group of congenital connective tissue disorders that are characterized by loose joints, joint pain, hyperelastic skin, and abnormal scar formation. These findings are often present during birth or early childhood. The syndrome results from mutations in genes involved in collagen synthesis. The classical type is notable for joint and skin symptoms and is caused by a mutation in type V collagen, while the vascular type is notable for vascular and organ ruptures and is caused by deficiencies in type III collagen. Patients often present with hypermobile joints that are unstable and prone to sprain, dislocation, subluxation, and hyperextension, skin that stretches beyond normal and tears and bruises easily, and joint disease. Other complications include aortic dissection, scoliosis, and chronic pain. Figure A is a clinical picture showing hyperelastic skin. Incorrect Answers: Answer 1: Child abuse may be confused with Ehlers-Danlos syndrome given the recurrent history of sprains and dislocations as well as findings (e.g., bruising) that may suggest abuse. However, this patient's hyperelastic skin findings and good eye contact suggest Ehlers-Danlos syndrome. Answer 2: Decreased production of type I collagen describes osteogenesis imperfecta, a genetic bone disorder that is characterized by multiple fractures with minimal trauma, blue sclerae, and opalescent teeth. Answer 3: Defective ATP7A leading to impaired copper absorption describes Menkes disease, which is characterized by brittle, kinky hair, growth retardation, and hypotonia. Answer 5: Mutation of the FBN1 gene describes Marfan syndrome, which has a lot of phenotypic overlap with Ehlers-Danlos syndrome. However, Marfan often affects the skeleton, heart, and eyes while Ehlers-Danlos (classical type) manifests with skin and joint symptoms. Bullet Summary: The classical subtype of Ehlers-Danlos syndrome, which manifests with skin and joint symptoms, is due to a mutation in type V collagen.
A 40-year-old male presents to your office complaining that he is too weak to climb stairs or brush his hair. He denies any headaches or change in vision. A muscle biopsy reveals CD8+ lymphocyte infiltration in the endomysium. Which of the following is the most likely diagnosis? A) Systemic lupus erythematosus B) Scleroderma C) Polymyalgia rheumatica D) Polymyositis E) Dermatomyositis
D, Polymyositis / Dermatomyositis This patient's presentation is consistent with a diagnosis of polymyositis. Polymyositis classically presents with hip and shoulder ("limb girdle") (i.e. proximal) weakness and atrophy. Polymyositis is an autoimmune disease in which T-cells infiltrate and damage muscle. On muscle biopsy, CD8+ lymphocyte destruction can be seen in the endomysium, the connective tissue that ensheaths each muscle fiber. Biopsy will also show overexpression of MHC class I on the sarcolemma, the muscle cell membrane underlying the endomysium. Koler et al. reviews a similar condition, dermatomyositis. In comparison to polymyositis, note that dermatomyositis presents with similar symptoms but with skin involvement, inflammation is present in the perimysium. Dermatomyositis is an antibody-mediated disease. Additionally, there is a strong association with an underlying malignancy which should warrant a full oncological workup. A review by Mammen considers the pathogenesis of polymyositis and dermatomyositis. Both conditions are autoimmune myopathies presenting with proximal muscle weakness, muscle inflammation, and extramuscular manifestations. Autoantibodies are frequently present. Although they are similar diseases, DM and PM are separate entities with different underlying pathophysiological mechanisms. Illustration A demonstrates the classic lymphocyte destruction of the endomysium (middle of the muscle fiber). Illustration B depicts characteristics that help differentiate normal muscle from polymyositis. Incorrect answers: Answer 1: Systemic lupus erythematosus is a multi-system autoimmune disease that usually affects women of childbearing age presenting with a wide variety of symptoms, including arthritis, malar rash, photosensitivity, renal disease, Libman-Sacks endocarditis, and more. Answer 2: Scleroderma is characterized by excessive production of collagen in the skin, GI tract, lungs, and kidneys. Answer 3: Polymyalgia rheumatica presents as joint pain and weakness in the hips and shoulders in elderly females; a typical polymyalgia rheumatica patient has trouble rising from a chair and lifting her arms over her head. Unlike polymyositis, muscle biopsy is unremarkable. Answer 5: Dermatomyositis presents in a similar fashion to polymyositis except that it also features dermatologic findings, such as a heliotrope rash.
A 38-year-old woman comes to the clinic complaining of joint pain for the past 3 months. She often wakes up with pain and stiffness for about 40 minutes. During this time, she is barely able to walk more than 5 steps due to the pain. However, the pain seems to alleviate as the day goes on. She works at a bar and reports that the back pain began several days after she lifted a case of beer. She denies any urinary incontinence, fevers, saddle anesthesia, weakness, or sensory changes. A physical examination demonstrates a skin rash shown in Figure A and joint tenderness of bilateral distal interphalangeal joints (DIP) and the left lower back. What finding would you expect in this patient? A) Bamboo spine B) Conjunctivitis C) Crypt abscesses on colonoscopy D) HLA-B27 positivity E) Rheumatoid factor positivity
D, Psoriatic Arthritis This patient has psoriatic arthritis (e.g., psoriatic lesions with arthritic pain), which has a strong association with HLA-B27. Psoriatic arthritis is one of the seronegative spondyloarthropathies, which is defined by arthritis without rheumatoid factor (anti-IgG antibody). Seronegative spondyloarthropathies include psoriatic arthritis, ankylosing spondylitis, inflammatory bowel disease, and reactive arthritis. Patients with psoriatic arthritis present with pain and stiffness with morning stiffness that lasts more than 30 minutes. Physical examination findings include stress pain, joint line tenderness, and effusion in joints in an asymmetric distribution. However, psoriatic arthritis may present with symmetric polyarthritis similar to, and at times indistinguishable from rheumatoid arthritis. Figure A is a clinical image of skin manifestations of psoriasis characterized by silver scales. Incorrect Answers: Answer 1: Bamboo spine, or vertical fusion of the spinal cord, is seen in ankylosing spondylitis. Although this patient is complaining of lower back pain, her skin lesions suggest psoriatic arthritis. Answer 2: Conjunctivitis can be seen in patients with reactive arthritis, which is characterized by conjunctivitis, urethritis, and arthritis. Answer 3: Crypt abscesses on colonoscopy are found in patients with inflammatory bowel disease. Patients often complain of abdominal pain and diarrhea; this patient is not presenting with these symptoms. Answer 5: Rheumatoid factor positivity would not be expected in this patient as psoriatic arthritis is a type of seronegative spondyloarthropathies. Bullet Summary: Psoriatic arthritis is associated with skin psoriasis and positive HLA-B27 in patients.
A 27-year-old man presents to the physician with concern for pain in both of his knees that he is unable to attribute to any activities that he has recently performed. He had an acute episode of diarrhea 2 weeks ago, prior to the onset of symptoms. He has also been experiencing a burning sensation during urination for the past week. When questioned about any other symptoms, he notes that he has also noticed that his eyes occasionally feel irritated and painful, leading to bouts of blurry vision. On physical examination, the patient is afebrile and has conjunctival injection surrounding the iris. A synovial fluid aspiration of the knee is performed and reveals a white blood cell count of 51,000/µL. Which of the following is the most appropriate treatment for this patient's condition? A) Acetaminophen B) Allopurinol C) Colchicine D) Indomethacin E) Leflunomide
D, Reactive Arthritis This patient with arthritis, urethritis, and uveitis most likely has reactive arthritis, which can be treated with indomethacin. Reactive arthritis is a type of arthritis that arises after an extra-articular infection (e.g., sexually-transmitted infection and gastroenteritis) and may cause arthritis symptoms such as joint pain and inflammation, urinary tract symptoms, as well as eye inflammation. Reactive arthritis is most commonly found in young males (20 to 40 years of age) with HLA-B27 positivity. Treatment involves medications to reduce joint pain and inflammation. Indomethacin, a non-steroidal anti-inflammatory drug (NSAID), can relieve the inflammation and pain of reactive arthritis. Incorrect Answers: Answer 1: Acetaminophen is an antipyretic and analgesic but does not have an anti-inflammatory mechanism of action and thus would not be indicated in the management of reactive arthritis. Answer 2: Allopurinol is a medication used in the treatment of gout, which is an inflammatory monoarthritis (most commonly in the big toe joint) caused by the precipitation of monosodium urate crystals in the joints. Answer 3: Colchicine is a medication used in the treatment of acute gout and familial Mediterranean fever (characterized by recurrent episodes of fever accompanied by pain in the abdomen, chest, or joints). Answer 5: Leflunomide is a medication used in the treatment of rheumatoid arthritis and psoriatic arthritis. Rheumatoid arthritis is characterized by pain, swelling, and stiffness in symmetric joints along with systemic symptoms including fever, fatigue, and weight loss. Psoriatic arthritis is characterized by joint inflammation along with scaly skin patches and nail lesions. Bullet Summary: Reactive arthritis is treated with non-steroidal anti-inflammatory drugs (NSAIDs).
A 35-year-old woman comes to your office with a variety of complaints. As part of her evaluation, she undergoes laboratory testing which reveals the presence of anti-centromere antibodies. All of the following symptoms and signs would be expected to be present EXCEPT: A) Pallor, cyanosis, and erythema of the hands B) Calcium deposits on digits C) Blanching vascular abnormalities D) Hypercoagulable state E) Heartburn and regurgitation
D, Scleroderma Anti-centromere antibodies are specific for limited systemic scleroderma (CREST). They are rarely found in other autoimmune disorders. Hypercoagulable state is not a feature of CREST, though it is a feature of autoimmune diseases such as anti-phospholipid antibody syndrome. Systemic sclerosis is an idiopathic connective tissue disease characterized by microvascular damage and overactive collagen deposition in skin and internal organs, with several subsets. CREST syndrome is a variant of limited cutaneous systemic sclerosis that includes Calcinosis of digits; Raynaud phenomenon; Esophageal dysfunction; Sclerodactyly; and Telangectasias. Anti-centromere antibodies are particularly associated with limited cutaneous sclerosis (including CREST). Anti-DNA-topoiomerase I(anti-Scl-70) antibodies are associated with diffuse sclerosis. Hinchcliff et al. describe the typical patient workup, to include bloodwork, pulmonary function tests, echocardiogram, and chest CT in order to establish the diagnosis and extent of visceral involvement. Spencer-Green et al. discuss the test characteristics of both anti-centromere antibodies and anti-DNA topoisomerase (anti-Scl-70) antibodies and conclude that both are highly specific for limited subsets of scleroderma but neither is sensitive enough to function as a screening test since 40% of systemic sclerosis patients will have neither antibody present. Illustration A shows an example of calcinosis of digits, specifically calcinosis cutis. Illustration B shows the pallor, cyanosis, and erythema of Raynaud phenomenon. Illustration C shows sclerodactyly. Illustration D shows telangiectasias distributed across the upper lip. Incorrect Answers: Answer 1: Pallor, cyanosis, and erythema of the hands describes Raynaud phenomenon, a component of CREST. Answer 2: Calcium deposits on the digits and extensor tendons are a feature of CREST. Answer 3: This describes telangiectasias found in CREST. Answer 5: Heartburn and regurgitation are symptoms of esophageal dysfunction in CREST.
A 47-year-old African American woman presents to her primary care physician with complaints of fatigue and joint stiffness for the past year. She reports that her joint stiffness is worse when she wakes up in the morning, but it is present throughout the day. Her temperature is 98.8°F (37.1°C), blood pressure is 146/92 mmHg, pulse is 86/min and regular, and respirations are 18/min. On physical exam the patient is noted to be wearing gloves, and she explains that her hands are often cold and painful. Examination of her hands without gloves shows her fingers are held in flexion bilaterally along with the findings in Figure A. Pulmonary examination is significant for bilateral inspiratory crackles. Which of the following is most likely to be additionally found in the patient? A) Bilateral hilar lymphadenopathy B) Erythema nodosum on the lower extremities C) Increased lower esophageal sphincter tone D) Lower esophageal atrophy and fibrosis E) Rheumatoid nodules on extensor surfaces
D, Scleroderma This patient's complaints of fatigue, joint stiffness, and cold hands coupled with the exam findings of sclerodactyly in Figure A, hypertension, and inspiratory crackles makes scleroderma the most likely diagnosis. Patients with scleroderma often have lower esophageal atrophy and fibrosis, leading to dilation and GERD. Scleroderma is a rheumatologic condition characterized by fibrosis of blood vessels, skin, and visceral organs. Its etiology is unknown. Patients often report fatigue and joint stiffness along with skin changes. Raynaud phenomena combined with sclerodactyly is often seen as well. Complications include pulmonary fibrosis which can present with inspiratory crackles as seen in this patient, along with hypertension due to renal involvement. Additionally, patients with scleroderma often have lower esophageal atrophy and fibrosis, leading to esophageal dilation and GERD. This complication is best managed with proton pump inhibitors. Figure A is a clinical image demonstrating sclerodactyly. Sclerodactyly typically presents with thickening of skin on the digits. As the disease progresses flexion contractures of the fingers are often seen. Incorrect Answers: Answer 1: Bilateral hilar lymphadenopathy is a finding that may be seen in sarcoidosis. Answer 2: Erythema nodosum on the lower extremities is an autoimmune panniculitis that manifests as tender nodules on the distal lower extremities. Erythema nodosum is associated with other autoimmune conditions such as inflammatory bowel disease and sarcoidosis, but it is not associated with scleroderma. Answer 3: Increased lower esophageal sphincter tone is a feature of achalasia. Achalasia is usually idiopathic, but may be associated with infection by Trypanosoma cruzi (Chagas disease). Answer 5: Rheumatoid nodules on extensor surfaces are features that are found in rheumatoid arthritis. They are often located within the patient's lungs or on the extensor surfaces of limbs. Bullet Summary: Patients with scleroderma often complain of heartburn secondary to lower esophageal atrophy and fibrosis.
A 29-year-old African American female presents to your office with extreme fatigue and bilateral joint pain. Serologies demonstrate the presence of rheumatoid factor along with anti-Smith and anti-dsDNA antibodies. A VDRL syphilis test is positive. You order a coagulation profile, which reveals normal bleeding time, normal PT, and prolonged PTT as well as normal platelet count. Further evaluation is most likely to reveal which of the following? A) Palmar rash B) HLA-B27 positivity C) Factor VIII deficiency D) History of multiple spontaneous abortions E) Immune thrombocytopenia
D, Systemic Lupus Erythematosus The patient described in the question stem has systemic lupus erythematosus (SLE). Ten to 30% of SLE patients have lupus anticoagulant, which can damage endothelium resulting in an increased incidence of thrombi, strokes, and spontaneous abortions (miscarriages). SLE patients can have a variety of autoantibodies present in the serum. Lupus anticoagulant and anticardiolipin antibodies are together referred to as antiphospholipid antibodies. Although lupus anticoagulant causes hypercoagulability, it paradoxically results in a prolonged PTT in laboratory tests. Lupus anticoagulant antibodies can result in a false positive VDRL test. Gill et al. discuss diagnosis of SLE. SLE is a connective tissue disease that affects the blood, joints, skin, and kidneys. It occurs most commonly in women of childbearing age. The clinical manifestations of SLE are wide-ranging and include malar rash, glomerulonephritis, pleuritis, pericarditis, photosensitivity, arthralgias, and more. Anti-Smith and anti-dsDNA antibodies are highly specific for SLE and are commonly used to diagnose the disease. Palomo et al. review antiphospholipid syndrome associated with antiphospholipid antibodies including lupus anticoagulant, anticardiolipin, and anti-Beta2-glycoprotein I antibodies. The history reveals one or more episodes of thrombosis as well as obstetric complications. Illustration A is a diagram showing pathogenesis of thrombosis in antiphospholipid antibody syndrome. Incorrect answers: Answer 1: This patient most likely does not have syphilis, rather the presence of lupus anticoagulant resulted in a false positive VDRL. Answer 2: HLA-B27 positivity is associated with psoriasis, ankylosing spondylitis, inflammatory bowel disease, and reactive arthritis (mnemonic: "PAIR"). It is not classically associated with SLE. Answer 3: A deficiency of Factor VIII is observed in hemophilia A and is not suggested by the history. Answer 5: Immune thrombocytopenia would be expected to present with isolated thrombocytopenia and would not result in increased PTT as seen with this patient.
A 31-year-old woman presents to her primary care physician for a 1-month history of intermittent fevers. She says that the fevers have been accompanied by fatigue as well as pain in her shoulders and knees bilaterally. Physical exam reveals a rash over her cheeks and the bridge of her nose as well as petechiae over her trunk. Her past medical history is significant for childhood asthma that has since resolved. She has not taken any medications for the last several years. Laboratory tests reveal a platelet count of 45,000/mm^3. Antibodies to which of the following would most likely also be seen in this patient? A) Deoxyribonucleic acid topoisomerase B) Helicase and synthetase C) Histones D) Small nuclear ribonucleoproteins E) U1 ribonucleoprotein
D, Systemic Lupus Erythematosus This patient who is experiencing intermittent fever, fatigue, arthralgias, malar rash, and thrombocytopenia most likely has systemic lupus erythematosus, which is associated with antibodies to small nuclear ribonucleoproteins. Systemic lupus erythematosus is a systemic autoimmune disease with flares that presents with fever, joint pain, and rash. The rash seen in a lupus flare can be a malar rash on the cheeks or discoid rashes with scaling. In addition to these acute manifestations, systemic effects of lupus include cardiovascular, hematologic, neurologic, and renal complications. Cardiovascular findings including pericarditis or myocarditis, hematologic findings include anemia or thrombocytopenia, neurologic findings include headaches or strokes, and renal complications include hematuria or proteinuria. Laboratory testing will reveal antinuclear antibodies, antibodies to double-stranded DNA, and antibodies to small nuclear ribonucleoproteins. Incorrect Answers: Answer 1: Antibodies to deoxyribonucleic acid topoisomerase are characteristic of scleroderma; however, this disease would present with skin tightening, Raynaud phenomenon, and esophageal dysmotility. Answer 2: Antibodies to helicase and synthetase would be seen in dermatomyositis, which can also present with a rash; however, this disease would present with proximal muscle weakness rather than fever and arthralgias. Answer 3: Antibodies to histones are highly specific for drug-induced lupus; however, this patient has not taken any medications in years and is unlikely to have a drug-induced etiology to her illness. Answer 5: Antibodies to U1 ribonucleoprotein are seen in mixed connective tissue disease, which can have all the features seen in lupus; however, this disease will have features from scleroderma and polymyositis as well. There are no findings from those other diseases in this patient so lupus is a more likely explanation for this patient's symptoms. Bullet Summary: Systemic lupus erythematosus flares will present with fever, arthralgias, and either a malar or a discoid rash.
A 38-year-old woman comes to clinic complaining of general malaise and a facial rash. She reports that the rash began one week ago after she had been playing outside with her dog. The rash has not resolved nor improved since then. She denies pain or pruritus. When asked about new exposures, the patient denies any changes in lotions or soaps, but does report she was recently prescribed a new medication by her primary care physician. The patient denies tobacco or illicit drug use. She is sexually active with one male partner. She works as a school nurse in an elementary school, and she recently traveled to Botswana for two months for a missionary trip. The patient's temperature is 99°F (37.2°C), blood pressure is 112/72 mmHg, pulse is 74/min, and respirations are 14/min with an oxygen saturation of 99% O2 on room air. On physical exam, you appreciate an oral ulcer, and mild joint stiffness and swelling of the patient's hands and knees. A photograph of the patient's facial rash is shown in Figure A. You feel her current presentation is drug-related. Which of the following could be the medication causing her symptoms? A) Amiodarone B) Carbamezapine C) Doxycycline D) Hydralazine E) Methotrexate
D, Systemic Lupus Erythematous This patient is presenting with malaise, a malar rash, and joint swelling that occurred following a recent drug exposure suggesting a diagnosis of drug-induced lupus. Hydralazine can precipitate drug-induced lupus. Drug-induced lupus is an autoimmune response following exposure to certain medications, which induces the production of autoantibodies. Common drugs that cause drug-induced lupus include quinidine, chlorpromazine, hydralazine, isoniazid, methyldopa, primaquine, penicillamine, and procainamide (mnemonic: Cute CHIMPPP). Patients with drug-induced lupus can develop a variety of systemic signs and symptoms resembling spontaneous lupus including fever, myalgias, rash, oral ulcers, arthralgias/arthritis, and serositis. Anti-histone antibodies are typically found in drug-induced lupus. Figure A shows a malar rash, which is characteristically red, mildly scaly, and spares the nasolabial folds. Incorrect Answers: Answer 1: Amiodarone can cause bluish pigmentation of the skin and photosensitivity (though less commonly). It can also cause hypothyroidism, peripheral neuropathy, pulmonary fibrosis, hepatotoxicity, and ocular symptoms. Answer 2: Carbamezapine can cause Stevens-Johnson syndrome (SJS). This is a severe, febrile desquamating/blistering disease of the skin, and always involves mucus membranes (e.g., ocular, oral, genital, and anal). Answer 3: Doxycycline can cause photosensitivity. Other adverse reactions include gastrointestinal distress. Answer 5: Methotrexate can cause photosensitivity as well as other oral and gastrointestinal mucosal symptoms (e.g., glossitis, entertitis, and mouth/gut ulcerations). Additional adverse effects include myelosuppression, hepatotoxicity, and pulmonary fibrosis. Bullet Summary: Drug-induce lupus can present with the classic symptoms of lupus following exposure to common causative agents, such as quinidine, chlorpromazine, hydralazine, isoniazid, methyldopa, primaquine, penicillamine, or procainamide.
A 22-year-old woman presents to her primary care provider complaining of a facial rash. She says the rash began 3 weeks ago after hiking in the White Mountains of New Hampshire this summer. Since that time she has also experienced pain in her hands and wrists that is worse in the morning and accompanied by subjective fevers. She denies chest pain, shortness of breath, nausea, or vomiting. Vital signs are 99.6°F (37.6°F), blood pressure is 134/82 mmHg, pulse is 88/min, and respirations are 18/min. Examination demonstrates a rash on the patient's face that spares the nasolabial folds along with oral ulcers. The metacarpophalangeal joints are tender to palpation, and range of motion is limited by pain. Complete blood count demonstrates normocytic anemia with thrombocytopenia. Which of the following is the next best step in diagnosis? A) Anti-cardiolipin antibodies B) Anti-dsDNA antibodies C) Anti-histone antibodies D) Anti-nuclear antibodies E) Anti-Smith antibodies
D, Systemic Lupus Erythematous This patient's presentation with malar rash, subjective fevers, joint pain and tenderness that is worse in the morning, and oral ulcers is consistent with systemic lupus erythematosus (SLE), which is best screened for with anti-nuclear antibodies. SLE is a multi-system disease that can cause renal disease, arthralgias, serositis, hematologic abnormalities, neurologic symptoms, immunologic derangements, and mucocutaneous lesions (malar and discoid rash, photosensitivity, and oral ulcers). Patients presenting with any combination of these symptoms should have SLE considered in their differential diagnosis. The most sensitive biomarker for lupus is titers for anti-nuclear antibodies (ANA). If positive, more specific antibodies such as anti-dsDNA and anti-Smith antibody testing should be obtained. Incorrect Answers: Answer 1: Anti-cardiolipin antibodies may be observed in SLE as well as a variety of other conditions such as antiphospholipid syndrome and syphilis. However, they are not sensitive for SLE, meaning there may be false negatives and are not a good initial test for ruling out SLE. Answer 2: Anti-dsDNA antibodies are commonly seen in SLE. They are associated with SLE nephritis, and their titers may correlate with disease activity. They are highly specific for SLE but are not as sensitive as anti-nuclear antibodies. Answer 3: Anti-histone antibodies are often observed in lupus, but they are not as sensitive as ANA. Anti-histone antibodies are however very sensitive for drug-induced lupus. Answer 5: Anti-Smith antibodies are highly specific for SLE; however, they are not sensitive. Anti-Smith antibodies should be obtained after a positive ANA result in patients with a clinical picture of SLE. Bullet Summary: Anti-nuclear antibody testing is the most sensitive test for systemic lupus erythematosus.
A 49-year-old woman with polymyositis presents to her physician's office for weakness. She reports that she feels unusually tired and weak. She works as a makeup artist and in the past few weeks has experienced difficulty applying makeup on her clients. She finds it difficult to work while she has her arms raised. She denies any fevers, weight gain or loss, or any other motor deficits. Physical exam reveals decreased strength in her shoulders. No rash is appreciated. Laboratory evaluation reveals increased creatinine phosphokinase, positive antinuclear antibody, and positive anti-signal recognition particle. She is started on high-dose corticosteroids. (Polymyositis)
Dermatomyositis / Polymyositis (snapshot)
A 31-year-old African American woman presents to her primary care provider complaining of stiff, painful fingers. She reports that her symptoms started 2 years ago and have gradually worsened. Her pain is not relieved by ibuprofen or acetaminophen. She is most concerned about having occasional episodes in which her fingers become extremely painful and turn white then pale blue. Her past medical history is notable for hypertension but she has previously refused to take any medication. She works as a postal worker and spends most of her time outside. Physical examination reveals induration of her digits with loss of skin fold wrinkles. She has limited finger range of motion. She would like to know if she can do anything to address her intermittent finger pain as it is affecting her ability to work outside in the cold. Which of the following medications is most appropriate to address this patient's concerns? A) Ambrisentan B) Diltiazem C) Enalapril D) Methotrexate E) Nifedipine
E, Scleroderma The patient in this vignette presents with finger induration, stiffness, and color change suggestive of scleroderma with associated Raynaud phenomenon. The first-line medical treatment for Raynaud phenomenon is a calcium channel blocker (CCB), frequently dihydropyridine CCBs such as nifedipine. Scleroderma is an autoimmune condition characterized by hardening and induration of the skin and/or other organs due to collagen deposition and fibrosis. Some patients with scleroderma may experience Raynaud phenomenon in which decreased blood flow to the skin from either cold temperature or stress causes vasospasms. Clinically, this presents as color change in the affected areas from white (ischemia) to blue (hypoxia) to red (reperfusion). Dihydropyridine CCBs such as nifedipine and amlodipine can be used to treat Raynaud phenomenon because these selectively reduce vascular tone. Incorrect Answers: Answer 1: Ambrisentan is an endothelin receptor antagonist that is used in combination with tadalafil (phosphodiesterase type 5 inhibitor) to treat pulmonary hypertension in patients with scleroderma. Answer 2: Diltiazem is a non-dihydropyridine CCB. Since non-dihydropyridine CCBs are less selective for peripheral arteries than dihydropyridine CCBs, they are not commonly used to treat Raynaud phenomenon. Answer 3: Enalapril is an angiotensin-converting enzyme (ACE) inhibitor that can be used to treat hypertension including in scleroderma patients who have hypertension due to renal involvement. Although this patient would likely benefit from an ACE inhibitor, these medications are not used to specifically treat Raynaud phenomenon. Answer 4: Methotrexate is an immunosuppressive agent that is one of the first-line treatments to prevent the progression of scleroderma. Although this patient would likely benefit from an immunosuppressive agent, methotrexate is not used to specifically treat Raynaud phenomenon. Bullet Summary: Dihydropyridine calcium channel blockers such as nifedipine or amlodipine can be used to treat Raynaud phenomenon in patients with scleroderma.
A 40-year-old woman presents to her primary care physician with complaints of worsening muscle aches and diffuse weakness over the past several months. She states that she now has difficulty climbing stairs and extending her neck. Ultimately, she became more concerned when she began having trouble swallowing and experienced shortness of breath last night which has resolved. Her temperature is 98.5°F (36.9°C), blood pressure is 120/84 mmHg, pulse is 70/min, respirations are 12/min, and oxygen saturation is 98% on room air. Physical exam is notable for 2/5 strength of the proximal muscles of the upper and lower extremities. Dermatologic, cardiac, and pulmonary exam are unremarkable. The patient is tolerating oral secretions and has no difficulty swallowing or protecting her airway currently. Laboratory studies are notable for an elevated CRP and aldolase. Which of the following is the most appropriate treatment for this patient? A) Intravenous immunoglobulin B) Methotrexate C) Naproxen D) Neostigmine E) Prednisone
E, Dermatomyositis / Polymyositis This patient's presentation is consistent with polymyositis given her proximal muscle weakness and elevated CRP/aldolase. The first-line treatment of inflammatory myositis is high-dose glucocorticoids (such as prednisone). Polymyositis is a systemic and inflammatory disease that is characterized by proximal muscle pain and weakness. More advanced disease is associated with shortness of breath and difficulty swallowing with involvement of the pharyngeal and/or esophageal musculature. Initial laboratory tests can include an ESR, CRP, and aldolase. EMG is abnormal in almost all patients with polymyositis. Muscle biopsy provides more definitive diagnosis and shows inflammation as well as muscle cell necrosis and regeneration. Management is first with corticosteroids followed by immunosuppressants and other agents (IVIG and monocolonal antibodies) as second-line agents as appropriate. Incorrect Answers: Answer 1: Intravenous immunoglobulin is indicated for steroid-resistant cases. First-line therapy should include steroids. It offers rapid control of symptoms and may be indicated for severe cases where the patient has dysphagia and cannot protect her airway (which is not occurring at this presentation). Answer 2: Methotrexate has limited data supporting its use in polymyositis in steroid-refractory cases. This DMARD may be indicated in rheumatoid arthritis which presents in middle-aged women with symmetric/proximal joint pain that is worse in the morning. Answer 3: Naproxen may reduce pain/stiffness; however, it will not treat the patient's underlying inflammatory condition and weakness. This may be indicated in the management of painful conditions such as polymyalgia rheumatica. Answer 5: Neostigmine/pyridostigmine are indicated in the treatment of myasthenia gravis which presents with muscle weakness/diplopia that is worse at the end of the day/after exertion. These medications are acetylcholinesterase inhibitors. Bullet Summary: Initial treatment of polymyositis is steroids.
A 42-year-old man presents to clinic complaining of increasing difficulty climbing stairs and standing up from sitting in his chair. On exam you perceive that his strength to be 5/5 distally, but only 3/5 in proximal muscle groups bilaterally. There is a distinctive rash on his upper eyelids and around his eyes. Examination of the fingers is most likely to reveal which of the following? A) Dactylitis B) Enlargement of the PIP C) Ulnar deviation of the fingers D) Nail pitting with oil spots E) Violaceous papules over the MCP, PIP, and DIP
E, Polymyositis / Dermatomyositis The patient's presentation is most consistent with dermatomyositis, an autoimmune disease characterized by proximal muscle weakness and cutaneous manifestations including heliotropic rash and Gottron papules. Gottron papules are violaceous, scaly patches that overlay the MCP, PIP, and DIP joints. The typical dermatomyositis patient is a woman in her 40s-60s who presents with hip or shoulder weakness and may have concurrent dysphonia or dysphagia. Patients with dermatomyositis show elevated serum creatinine kinase and muscle biopsy shows lymphocytic infiltrate. Serum ANA is positive <30% of the time and is non-specific, anti-Jo Antibodies are a more specific marker. Patients who develop dermatomyositis over the age of 50 should warrant increased clinical suspicion for underlying malignancy. Koler et al. review dermatomyositis. While the etiology of dermatomyositis remains unknown, it is frequently seen in individuals with other connective tissue disorders and can signal the presence of an underlying malignancy. Disease management includes: evaluation to rule out malignancy, systemic corticosteroids, and sunscreen for skin manifestations. Polymyositis is a similar idiopathic inflammatory myopathy that does not have skin involvement. Robinson et al. review the manifestations of dermatomyositis in the pediatric population. While there is some overlap in between adult and juvenile dermatomyositis, they are considered separate entities with distinct features. Juvenile dermatomyositis presents with more vasculopathy than adult dermatomyositis. Additionally, juveniles are also less likely to develop interstitial lung disease or having associated malignancy. Illustration A shows a pathologic section from muscle biopsy in a dermatomyositis patient highlighting the inflammtory infiltrate around muscle fibers. Incorrect Answers Answer 1: Dactylitis or "sausage fingers" is associated with psoriatic arthritis. Answer 2: Enlargement of the PIP, also known as a Boutonniere deformity, is often seen in rheumatoid arthritis. Answer 3: Ulnar deviation is often seen in rheumatoid arthritis. Answer 4: Nail pitting with oil spots are characteristically seen in patients with psoriasis.
A 28-year-old man presents with one week of redness and discharge in his eyes, pain and swelling in his left second and third toes, and rash on the soles of his feet. He is sexually active with multiple partners and uses condoms occasionally. He denies any recent travel or illness and does not take any medications. Review of systems is otherwise unremarkable. On physical exam, he has bilateral conjunctivitis, dactylitis of the left second and third toes, and crusty yellow-brown vesicles on his plantar feet. Complete blood count and chemistries are within normal limits. Erythrocyte sedimentation rate (ESR) is 40 mm/h. Toe radiographs demonstrate soft tissue swelling but no fractures. Which diagnostic test should be performed next? A) Rheumatoid factor B) Anti-cyclic citrullinated peptide antibody assay C) Antinuclear antibody assay D) HLA-B27 E) Nucleic acid amplification testing for Chlamydia trachomatis
E, Reactive Arthritis This clinical presentation is most consistent with reactive arthritis, likely due to Chlamydia infection. Therefore, nucleic acid amplification testing for Chlamydia trachomatis should be performed. Chlamydia infection is a common cause of reactive arthritis (formerly known as Reiter's syndrome). The classic triad of symptoms are urethritis, conjunctivitis, and asymmetric non-infectious arthritis. Associated skin findings include circinate balanitis, a dry annular rash involving the glans and shaft of the penis, and keratoderma blenorrhagica, hyperkeratotic, sometimes pustular skin lesions involving the palms and soles. Suspected patients should be tested for Chlamydia via nucleic acid amplification testing. Infected patients can be treated with rifampin and doxycycline or azithromycin. Incorrect Answers: Answer 1: Rheumatoid factor is associated with rheumatoid arthritis. Answer 2: Anti-cyclic citrullinated peptide antibodies are also associated with rheumatoid arthritis and more specific than rheumatoid factor. Answer 3: Antinuclear antibody assay is a non-specific test, associated with systemic lupus erythematosus, scleroderma, Sjogren's syndrome, and mixed connective tissue disease. Answer 4: While HLA-B27 positivity may predispose to reactive arthritis, it is not the best answer choice since it will not influence the management of this patient. Bullet Summary: The classic triad of symptoms of Reiter's syndrome is urethritis, conjunctivitis, and asymmetric non-infectious arthritis.
A 40-year-old Caucasian female presents to your office with fever, fatigue, cold fingers, and the rash shown in Figure A. The presence of which of the following antibodies has the highest specificity for diagnosis of this patient's condition? A) Anti-acetylcholine receptor B) Anti-Ro C) Anti-La D) Anti-ssDNA E) Anti-dsDNA
E, Systemic Lupus Erythematosus The patient's presentation is consistent with systemic lupus erythematosus (SLE). Anti-dsDNA and Anti-Smith antibodies have the greatest specificity, 95% and 99% respectively, for diagnosing SLE. SLE is a connective tissue disease that affects vasculature, joints, skin, and kidneys. It occurs most commonly in women of childbearing age. The clinical manifestations of SLE are highly variable and include: malar rash, glomerulonephritis, pleuritis, pericarditis, photosensitivity, arthralgias, and more. Gill et al. review the use of anti-dsDNA antibodies in the diagnosis of SLE. In order to make the diagnosis, 4 of 11 clinical and laboratory criteria must be met. The primary laboratory test is an antinuclear antibody titer, however given the low prevalence of SLE in the general population, the antinuclear antibody titer has a relatively low predictive value in asymptomatic individuals. The American College of Rheumatology recommends obtaining a titer only in individuals present with characteristic symptoms of at least two organ systems. Munoz et al. discuss the role of anti-dsDNA antibodies in the pathogenesis of SLE. Anti-dsDNA is highly specific for SLE and can be found in 95% of individuals with active, untreated disease. The authors posit that biologic and pathogenic properties of Anti-dsDNA is directly responsible for some of the organ damage seen in SLE patients. Figure A depicts a malar rash in the characteristic butterfly pattern seen in SLE patients. Illustration A depicts an algorithm for diagnosing SLE. Incorrect answers: Answer 1: Anti-ACh receptor antibodies are seen in myasthenia gravis. Answer 2: Anti-Ro (Anti-SS-A) antibodies are most specific for Sjogren's syndrome (70-95%) and have a 25-50% specificity for SLE. Answer 3: Anti-La (Anti-SS-B) antibodies have a 60-90% sensitivity for Sjogren's syndrome. Answer 4: Anti-ssDNA antibodies are not used to diagnose SLE.
A 27-year-old female is brought to the emergency department by police when she attacked her date outside of a pizza restaurant. The patient refused to explain her behavior to the police. She seemed to be responding to internal stimuli when being initially interviewed. The patient has a past medical history of depression, anxiety, and obesity. The patient has a known history of medication non-compliance. Her temperature is 99.5°F (37.5°C), pulse is 112/min, blood pressure is 100/70 mmHg, respirations are 18/min, and oxygen saturation is 98% on room air. On physical exam the patient is poorly kempt and has an odd body odor. Inspection of her oropharynx reveals multiple ulcers. Cardiac exam is unrevealing with the exception of a friction rub. Since the incident, the patient has calmed down and is currently resting. She is no longer responding to internal stimuli and is more cooperative. Lab values are obtained and are below. Serum: Na+: 135 mEq/L Cl-: 100 mEq/L K+: 3.9 mEq/L HCO3-: 24 mEq/L BUN: 20 mg/dL Glucose: 70 mg/dL Creatinine: 1.7 mg/dL Ca2+: 10.0 mg/dL Mg2+: 1.5 mEq/L Hemoglobin: 10 g/dL Hematocrit: 35% Reticulocyte count: 4.9% Platelet count: 275,000/mm^3 Leukocyte count: 5,500/mm^3 Which of the following is the next best step in management for this patient's underlying condition? A) IM haloperidol B) Haloperidol depot C) Clozapine D) Urine toxicology E) ANA and anti-dsDNA levels
E, Systemic Lupus Erythematous This patient is presenting with 5 of 11 criteria for systemic lupus erythematosus (SLE). The best next step in management for this patient's underlying condition is an ANA and anti-dsDNA level to support the diagnosis. The diagnosis of SLE can be made when 4 of 11 SLE criteria are met (serositis (pleuritis/pericarditis), oral ulcers, arthritis, photosensitivity, blood disorder (anemia, leukopenia), renal problems, ANA (+), immunological (anti-dsDNA or anti-Smith), neurological symptoms (psychosis, seizures), malar rash, discoid rash). This patient is presenting with serositis (pericardial friction rub), oral ulcers, hemolytic anemia, new-onset psychosis, and renal failure. To further confirm the diagnosis, a sensitive test (ANA titer) and specific test (anti-dsDNA titer or anti-Smith titer) should be obtained prior to the onset of therapy. Incorrect Answers: Answer 1: IM haloperidol would offer quick-onset relief of this patient's new-onset psychosis, in particular if the patient refuses to take any medications by mouth. However, this patient is no longer experiencing any psychotic symptoms (she is resting in bed peacefully) and it would be more appropriate to further confirm the diagnosis first. Answer 2: Haloperidol depot is the treatment of choice for schizophrenic patients who do not reliably take their medications as it offers a long-acting therapy to control their symptoms. Since this patient no longer seems to be experiencing symptoms of psychosis, further workup is a more appropriate next step. Answer 3: Clozapine is the most effective anti-psychotic that is typically used as a final line in treatment of schizophrenia due to its side-effects such as agranulocytosis. This would not be used first-line to manage psychosis, and further workup should be explored to confirm the diagnosis. Answer 4: Urine toxicology would be an appropriate initial step in workup in a patient presenting with new-onset psychosis. This question asked which of the following is the best next step in management for this patient's underlying condition. Since this patient's underlying condition is SLE, further confirming the diagnosis is a better answer. Bullet Summary: The best initial step in management is to confirm the diagnosis with an ANA level (sensitive test) and anti-dsDNA or anti-Smith level (specific test).
An 8-year-old boy is brought to his pediatrician for developing multiple bruises over the course of the school year. The patient's mother believes the child is being bullied at school. The patient is otherwise healthy. Family history is significant for his father expiring at 38 yearas of age due to a severe subarachnoid hemorrhage. On physical exam, there are ecchymoses and hematomas over the shins and knees. The skin also appears fragile and thin, which demonstrates a venous pattern in the extremities. The child undergoes genetic testing to confirm the diagnosis.
Ehlers-Danlos Syndrome (snapshot)
A 53-year-old woman with a past medical history of thyroid disease presents to her physician's office for weakness. She reports that she has been feeling weak, has difficulty climbing the stairs, and combing her hair every morning. Today, she also has difficulty arising out of her chair. Physical exam reveals decreased strength, especially in the shoulders. She also has an impressive lilac periorbital rash and a sunburn on her cheeks. She is sent for further laboratory workup and counseled on the likely initiation of high-dose steroids.
Polymyositis / Dermatomyositis (snapshot)
A 32-year-old man presents to his rheumatologist for evaluation of new-onset joint pain. He reports feeling stiffness in his lower back in the morning and in the most distal joints in his left hand. He has a past medical history of psoriasis. On physical exam, he has papules and plaques along his hairline and scalp with overlying silver scales. His left middle finger has dactylitis and the distal interphalangeal joint is swollen. Laboratory evaluation shows that he is seronegative for rheumatoid factor. He is started on apremilast.
Psoriatic Arthritis (snapshot)
A 45-year-old woman presents to her physician with swollen fingers, joint pains, and a dry cough. She reports that this started a year ago and has not improved. She has a past medical history of vitiligo and primary biliary cholangitis. Physical exam reveals tightened, shiny skin with induration over her face and arms, sclerodactyly, and dry rales in the lungs. There are also telangiectasias on her left cheek. Her physician sends her for additional pulmonary imaging and autoimmune workup.
Scleroderma (snapshot)
A 50-year-old man presents to his physician's office for discolored fingers. He reports that this happens in the cold or when he is emotionally stressed. As he is going through a divorce, he reports this happening more often than not. He reports that there is no pain, but he sometimes feels tightening in his fingers with the discoloration. He has a past medical history of limited scleroderma as well as a family history of Raynaud phenomenon. His physical exam is notable for blue fingertips. He is prescribed a calcium channel blocker for his symptoms.
Raynaud Phenomenon (snapshot)
A 22-year-old man presents to his primary care physician's office due to pain in his knee, dysuria, and eye discomfort and discharge. Medical history is significant for a sexually transmitted infection due to Chlamydia approximately 1 month ago. On physical exam, there is conjunctivitis, left-knee swelling and tenderness, and urethral discharge.
Reactive Arthritis (snapshot)
A 32-year-old man presents with redness of the eye as well as discharge. He reports that he experiences pain with urination and stiffness and pain of the knee and ankle. He says that he had a sexually transmitted disease approximately 5 weeks ago but is otherwise healthy. On physical exam there is conjunctivitis, asymmetric oligoarthritis, and discharge from the urethral meatus.
Reactive Arthritis (snapshot)
A 50-year-old woman presents to her physician's office for a routine checkup. Once at the office, she reports that she has been generally doing well but recently noticed that her fingers tend to turn blue in the cold. She reports feeling a general skin tightening in her face and hands, which makes forming a fist difficult. She also notes that she has had increased acid reflux lately and requests a medication for that. Her past medical history includes autoimmune thyroid disease and alopecia areata. Physical exam reveals sclerodactyly, and tight, hardened skin with limited mobility in her fingers. Her physician sends her for additional autoimmune workup.
Scleroderma (snapshot)
A 45-year-old male with no significant medical history comes to your office complaining of pain and stiffness in his hands upon wakening in the morning for the past several months. The stiffness generally lasts one to two hours, per the patient. You perform an x-ray (Figure A). What is the most likely diagnosis? A) Gouty arthritis B) Osteoarthritis (OA) C) Rheumatoid arthritis (RA) D) Psoriatic arthritis E) Systemic lupus erythematosus (SLE)
The morning stiffness lasting more than one hour and radiographic findings demonstrating joint space narrowing and erosion of the proximal interphalangeal joints (PIPs) makes rheumatoid arthritis (RA), answer 3, the most likely. RA is a symmetric, inflammatory polyarthritis. RA should be considered in any patient presenting with joint pain and morning stiffness at least for one hour before maximal improvement for an extended period of time (usually >6 weeks). The diagnosis of RA is confirmed with imaging, positive rheumatoid factor (RF) and/or anti-citrullinated peptide antibody (anti-CCP), and elevated C-reative protein or erythrocyte sedimentation rate. As Wasserman notes, caution should be used with interpreting RF results. RF is not specific for RA and can be positive in individuals with other diseases (e.g. hepatitis C) or in healthy older individuals. Aletaha et al. outline specific criteria for the diagnosis of RA, which was most recently established by the American College of Rheumatology and European League Against Rheumatism in 2010. These guidelines were established in an effort to diagnose RA earlier in patients who may not meet the previously used guidelines (the 1987 American College of Rheumatology classification criteria). Figure A shows a radiograph of a person with RA. Illustration A is a table outlining key differences in RA vs. OA. Illustration B is a table outlining characteristics of different arthritides. Illustration C shows the 2010 American College of Rheumatology/European League Against Rheumatism Classification Criteria for RA. Illustration D lists extraarticular manifestations of RA. Illustration E shows the hands of a person with RA. Note the swelling in the MCP joints, but not the DIP joints. Also, note the ulnar deviation, which is another common finding in RA. Incorrect Answers: Answer 1: Gouty arthritis is usually monoarticular and tophi may appear on imaging. Answer 2: Morning stiffness in OA lasts less than an hour. Answer 4: Psoriatic arthritis shows a pencil-in-a-cup deformity on imaging. Answer 5: SLE would most likely present with many other constitutional symptoms (e.g. malar rash, photosensitivity, etc.) and is less common than RA. A 21-year-old man presents with severe morning back pain over the past three months. His pain improves as the day progreses and with excercise. Physical exam shows a stooped posture and diminished anterior flexion of the lumbar spine. Radiography of the lumbar spine shows bilateral sclerotic changes in the sacroiliac area. Laboratory testing shows that the patient is HLA-B27 positive.
A 26-year-old woman presents to the dermatology clinic for fatigue, weakness, and fevers for the past month. She reports significant weight loss despite eating a normal diet. She reports that she sunburns very easily and has a facial rash that is hard to cover with makeup. On physical exam, she has a butterfly rash with nasolabial sparing on her face, several discoid lesions on her fingers, and a erythematous rash on her chest in a V-neck distribution. On laboratory exam, she has a highly positive antinuclear antibody and positive anti-double-stranded DNA antibody.
Systemic Lupus Erythematosus (snapshot)
A 46-year-old African American woman with history of systemic lupus erythematosus presents with increasing fatigue, arthralgias, rash, and hematuria. She reports that she had previously been on hydroxychloroquine with success but had not seen a physician in 5 years. Physical exam shows a malar rash on her face. Laboratory exam reveals a highly positive antinuclear, anti-double-stranded DNA, and anti-Smith antibodies. She also has low complement levels. She is restarted on hydroxychloroquine and steroids.
Systemic Lupus Erythematous (snapshot)
A 42-year-old male comes to your clinic complaining of dizziness and shortness of breath. He says his symptoms started this morning while he was helping his girlfriend move out of her apartment. If he had not sat down, the patient reports he "would have fainted." His review of systems is notable for fatigue and lower back pain. His past medical history is significant for psoriasis treated with topical steroids, and asthma, for which he uses a short-acting, albuterol inhaler. He tried using his inhaler without notable improvement. His temperature is 99°F (37.2°C), blood pressure is 102/68 mmHg, pulse is weak at 50/min, and respirations are 24/min with an oxygen saturation of 95% O2 on room air. On physical exam, you note mild bilateral lower extremity edema. An EKG is performed, as shown in Figure A. The patient is given a permanent ventricular pacemaker. During a one month follow-up visit, he reports improvement of his cardiopulmonary symptoms, but complains of persistent lower back pain. He reports he has had back pain for at least six months, but believes it got worse after he had helped his girlfriend move out of her apartment prior to his initial presentation to the clinic. He reports he "can hardly get out of bed in the morning." What is the best initial diagnostic test for this patient's condition? A) Plain radiograph B) Magnetic resonance imaging C) Computerized tomography angiography D) Erythrocyte sedimentation rate E) HLA-B27 test
The patient is presenting with a 3rd degree atrioventricular heart block and lower back pain that is worse in the morning, suggesting the diagnosis of ankylosing spondylitis (AS). The best initial diagnostic test is a plain radiograph. Ankylosing spondylitis is associated with HLA-B27, which has been shown to have a relationship with cardiac complications including AV block or bundle branch blocks. The workup for this patient should include the best initial diagnostic test, a plain radiograph, which may show the classic "bamboo spine" appearance caused by vertical joint fusion and sclerotic changes in the sacroiliac joint (Illustration A). This would be followed by the most accurate diagnostic test, which is an MRI of the sacroiliac joint. Treatment for 3rd degree heart block almost always requires a pacemaker. Treatment for AS involves indomethacin for pain management and TNF inhibitors or sulfasalazine for severe/refractory disease in addition to physical therapy. Figure A shows an EKG of 3rd degree heart block, exhibiting the characteristic atrial-ventricular dissociation. Illustration A is a radiograph showing fusion of multiple disc spaces and squaring of the vertebral bodies. Incorrect Answers: Answer 2: While MRI of the sacroiliac joint is the most accurate diagnostic test for ankylosing spondylitis, is it not the best initial test. An MRI would also be a good initial test for a patient showing symptoms indicating neurologic deficiencies such as saddle anesthesia, motor deficits, or bladder/bowel incontinence, none of which this patient complains of. Answer 3: CT angiography would be a good diagnostic test for vascular intermittent claudication. While the patient has cardiovascular risk factors, and lower back pain can be a manifestation of proximal vessel peripheral arterial disease, the discomfort usually occurs with activity and is alleviated by rest. Answer 4: ESR can be elevated in AS, as it is an indicator of inflammation. However, it is neither sensitive nor specific, and thus is a poor initial diagnostic test. It is estimated that less than 70% of people with AS have an increased ESR level. In addition, an elevated ESR level can be seen with infection or cancer, neither of which the patient shows risk factors for. Answer 5: While a majority of patients with AS will test positive for HLA-B27, it is not diagnostic. Additionally, the association between AS and HLA-B27 varies by ethnic and racial groups. Bullet Summary: The best initial diagnostic test for ankylosing spondylitis is a plain radiograph, while the most accurate diagnostic test is an MRI of the SI joint.
A 42-year-old painter presents to the clinic with worsening arthritis. She reports that for the past 2 years she has experienced pain, swelling, and morning stiffness within the joints of her hands. The patient states that this has resulted in decreased grip strength, which is affecting her work. The patient's only medication is ibuprofen. Physical examination shows hand deformities. The patient's right hand is pictured in Figure A. Which of the following antibodies is the most specific for this patient's current condition? A) Anti-CCP antibody B) Anti-centromere antibody C) Antinuclear antibody D) Anti-Smith antibody E) Rheumatoid factor
The patient is presenting with hand stiffness, joint swelling, and evidence of boutonniere deformity, which suggests the diagnosis of rheumatoid arthritis (RA). Anti-CCP is the most specific antibody for RA. Rheumatoid arthritis (RA) presents with morning stiffness, pain and swelling of the metacarpophalangeal (MCP) and proximal interphalangeal (PIP) joints as well as decreased grip strength. Some classic physical exam findings include subcutaneous rheumatoid nodules, ulnar deviation of the fingers (Illustration A), swan neck deformity (distal interphalangeal joint (DIP) flexed while PIP is hyperextended) and boutonniere deformity (PIP joint flexed while DIP is hyperextended). The most specific antibody for RA is anti-CCP. Figure A illustrates a boutonniere deformity of the thumb, early ulnar deviation, and joint swelling, seen in RA. Illustration A illustrates late-stage RA of the hand with severe ulnar deviation, joint swelling, and boutonniere deformity. Incorrect Answers: Answer 2: Anti-centromere antibody is a specific antibody for limited scleroderma or CREST syndrome. Answer 3: Antinuclear antibody is a highly sensitive antibody for systemic lupus erythematosus but is largely non-specific. Answer 3: Anti-Smith antibody is the most specific antibody for systemic lupus erythematosus. Answer 4: Rheumatoid factor can be seen in elevated levels in RA but is non-specific. Bullet Summary: Anti-CCP antibody is the most specific antibody for rheumatoid arthritis.
A 34-year-old woman presents to your clinic with a 6-month history of pain in her hands. The pain is worse in the morning but is present throughout the day despite her use of ibuprofen. Additionally, her left 4th finger recently began to swell and is particularly painful. She has also begun to notice changes in her nails, which have become brittle. Prior to the onset of symptoms, she made multiple visits to the clinic with complaints of pain in her lower back and ankles that was eventually well controlled with ibuprofen. Her temperature is 99.5°F (37.5°C), pressure is 138/88 mmHg, pulse is 78/min, and respirations are 18/min. Inspection of the hands demonstrates the physical examination findings present in Figure A. Additionally, a rash on the right elbow is identified and pictured in Figure B. The distal interphalangeal joints of the left hand are tender to palpation, as is the second distal interphalangeal joint of the right hand. The Achilles tendons are tender bilaterally, and ultrasonography is consistent with mild Achilles tendonitis bilaterally. Testing for HLA-B27 is positive. Which of the following is the most likely diagnosis? A) Ankylosing spondylitis B) Gout C) Osteoarthritis D) Psoriatic arthritis E) Rheumatoid arthritis
The presence of a positive HLA B27 test with nail pitting, dactylitis, enthesitis, pain and tenderness of the distal interphalangeal (DIP) joints, and a rash consisting of a salmon-colored plaque with silver scaling is consistent with a diagnosis of psoriatic arthritis (PA). Psoriatic arthritis occurs in up to one-third of patients with psoriasis and is associated with HLA B27. The DIP joints are the most commonly involved, but patients may also have sacroiliitis and enthesitis at presentation combined with the classical rash of psoriasis and nail pitting. Severe cases may also have dactylitis resulting in the classic "sausage" fingers and toes. The first-line treatment for PA is nonsteroidal anti-inflammatory drugs (NSAIDs), but refractory or severe disease should be treated with either disease modifying anti-rheumatic drugs or biologics. Figure A demonstrates the dactylitis and nail pitting commonly seen in psoriatic arthritis. Figure B demonstrates a salmon-colored plaque with silver scaling, characteristic of psoriasis and psoriatic arthritis. Incorrect Answers: Answer 1: Ankylosing spondylitis is an HLA-B27 associated syndrome and may share features with PA. While sacroiliitis and enthesitis (particularly of the Achilles tendons) are common in ankylosing spondylitis, they also may be observed in PA. The presence of the characteristic rash of psoriasis and nail-pitting makes a diagnosis of PA more likely. Answer 2: Gout often presents with arthritis and cutaneous findings such as tophi. This patient lacks tophi and instead has nail pitting and the characteristic silver scaling salmon-colored plaques of psoriasis, making PA more likely. Answer 3: Osteoarthritis often affects the DIP joints. While this patient has tenderness of the DIPs, the presence of nail pitting, silver scaling salmon-colored plaques, and dactylitis makes a diagnosis of PA more likely. Answer 5: Rheumatoid arthritis is a common cause of arthritis. However, the proximal interphalangeal joints are preferentially affected, and dactylitis would be highly unusual. This combined with the aforementioned cutaneous findings makes a diagnosis of PA more likely in this patient. Bullet Summary: Psoriatic arthritis classically presents with enthesitis, arthritis of the distal interphalangeal joints, nail pitting, and salmon-colored plaques with silver scaling.
A 45-year-old male with no significant medical history comes to your office complaining of pain and stiffness in his hands upon awakening in the morning. He says the pain and stiffness tends to go away an hour or so after waking up. On physical exam, you note scaly rashes on his elbows bilaterally. You obtain radiographs, shown in Figure A. What is the most likely diagnosis? A) Gouty arthritis B) Osteoarthritis C) Rheumatoid arthritis D) Psoriatic arthritis E) Systemic lupus erythematosus
The radiograph of both hands demonstrates pencil-in-cup deformities of both thumbs and erosion of the DIP joint of left middle finger, making psoriatic arthritis (in the context of the scaly rashes on the bilateral elbows) the most likely diagnosis. Psoriatic arthritis is an inflammatory arthritis associated with psoriasis. Although it most commonly presents in patients who already have documented psoriasis, up to 20% of patients develop arthritis before the development of psoriatic skin lesions. The characteristic findings in psoriatic arthritis include distal interphalangeal (DIP) joint involvement (often with osteolysis resulting in the classic pencil-in-cup deformity), asymmetry, dactylitis (known commonly as "sausage digits"), flail or ankylotic deformities of the digits, and the frequent presence of enthesitis and spinal involvement. Weigle et al. review psoriasis and its associated with systemic manifestations. They write that psoriasis is associated with several non-skin disorders including arthritis, cardiovascular disease, lymphoma, and depression. Reveille et al. discuss the extensive clinical overlap between psoriatic arthritis, psoriasis, and Reiter's syndrome. Based on the results of their study, they hypothesized that some forms of psoriatic arthritis (e.g. HIV-associated) may constitute a spondlyarthropathy more closely related to reactive arthritis (also known as, Reiter's syndrome). Figure A shows the characteristic findings of psoriatic arthritis on radiographs (pencil-in-cup deformities of both thumbs and erosion of the DIP joint of left middle finger). Incorrect Answers: Answer 1: Gouty arthritis is usually a monoarticular arthritis and does not show pencil-in-cup deformities. Answer 2: Osteoarthritis is associated with morning stiffness that lasts less than an hour; it is also associated with different findings on imaging (e.g. joint space narrowing, osteophytes, and subchondral sclerosis). Answer 3: RA does not affect the DIP joints and does not show cup-and-pencil deformities on imaging. Also, RA frequently presents with other constitutional symptoms. Answer 5: Arthritis associated with systemic lupus erythematosus is generally associated with other constitutional symptoms.
A 28-year-old woman presents to the physician's office with symmetric joint pains throughout her body. She states that she has never had these symptoms before, but recently began experiencing symmetric joint pain in her upper extremities that is worse in the morning and improved with exercise. Her joint pain is most notable in her hands and wrists. The patient is otherwise generally healthy. She states that she lives at home with her husband and son. She is stressed because her son is currently sick with a fever and has to accompany her throughout her daily routine (Figure A). Her mother has a history of rheumatoid arthritis, and her father suffers from osteoarthritis. Her temperature is 98.3°F (36.8°C), blood pressure is 122/82 mmHg, pulse is 80/min, respirations are 15/min, and oxygen saturation is 99% on room air. Physical exam including cardiopulmonary and dermatologic exam is unremarkable. Which of the following is the most likely etiology of this patient's symptoms? A) Anti-dsDNA positive condition B) Localized bacterial infection C) Mechanical cartilage damage D) Systemic viral infection E Type IV hypersensitivity reaction
This patient has with a child with a likely parvovirus B19 infection (slapped cheeks and a fever) and also new-onset symmetric arthralgias, suggesting she also has this infection. Parvovirus B19 can cause erythema infectiosum in pediatric patients, which presents with slapped cheeks and a self-limiting fever. On the other hand, in adults, symptoms are often nonspecific and may present with only symmetric arthralgias that resolve on their own in weeks to months. The diagnosis is generally made clinically; however, serology can be used to confirm the diagnosis in uncertain cases. Treatment is supportive (NSAIDs and oral fluids). A unique, life-threatening complication of a parvovirus infection is aplastic anemia, which is usually clinically apparent in patients who typically lose red blood cells at a more rapid rate (such as patients with hereditary spherocytosis or sickle cell disease). These patients require transfusions until the infection resolves. Figure A is a child with slapped cheeks in the setting of a fever, which is suggestive of parvovirus B19 infection. Incorrect Answers: Answer 1: Anti-dsDNA positive condition represents systemic lupus erythematosus (SLE). Symmetric arthralgias are the most common initial presenting symptoms for SLE; however, this patient has an exposure to someone with parvovirus B19 with no other history or symptoms supportive of a diagnosis of SLE, making this a less likely diagnosis. Answer 2: Localized bacterial infection describes septic arthritis, which presents with a red, hot, inflamed joint that is exquisitely tender to movement and is associated with an elevated ESR/CRP. Answer 3: Mechanical cartilage damage describes osteoarthritis, which is typically seen in a weight-bearing joint of an obese patient with symptoms worsened by weight-bearing and relieved by rest. The joint is often cold, not inflamed/warm, and symptoms are worsened with exertion. Answer 5: Type IV hypersensitivity reaction describes rheumatoid arthritis, which commonly presents in a young woman with symmetric arthralgias. Though this diagnosis is certainly possible, this woman has no other symptoms of rheumatoid arthritis (which may not be necessary as this could be her initial presentation) and has exposure to a patient with parvovirus. Bullet Summary: Parvovirus B19 presents with slapped cheek fever in pediatric patients and symmetric arthralgias in adults.
A 27-year-old man presents to the emergency department with back pain. The patient states that he has back pain that has been steadily worsening over the past month. He states that his pain is worse in the morning but feels better after he finishes at work for the day. He rates his current pain as a 7/10 and says that he feels short of breath. His temperature is 99.5°F (37.5°C), blood pressure is 130/85 mmHg, pulse is 80/min, respirations are 14/min, and oxygen saturation is 99% on room air. On physical exam, you note a young man who does not appear to be in any distress. Cardiac exam is within normal limits. Pulmonary exam is notable only for a minor decrease in air movement bilaterally at the lung bases. Musculoskeletal exam reveals a decrease in mobility of the back in all four directions. Which of the following is the best initial step in management of this patient? A) Pulmonary function tests B) Radiography of the lumbosacral spine C) MRI of the sacroiliac joint D) CT scan of the chest E) Ultrasound
This patient is a young male presenting with back pain that improves upon activity and decreased mobility suggesting a diagnosis of ankylosing spondylitis (AS). The best initial step in management of AS is radiography of the lumbosacral spine to look for sacroiliitis. Ankylosing spondylitis classically presents in a young male who complains of back pain that improves with activity. Patients may experience other complications including uveitis, aortitis, and restrictive lung disease. The best initial step in management is a radiograph of the lumbosacral spine to look for signs of sacroiliitis which is typically the first radiographical finding. The most accurate test is an MRI of the sacroiliac joint. Illustration A is an MRI demonstrating sacroiliitis. This is a T1-weighted image before and after contrast injection with the arrow identifying inflammation at the SI joint. Incorrect Answers: Answer 1: Pulmonary function tests could be used to diagnose restrictive lung disease which this patient may be experiencing given his shortness of breath. Restrictive lung disease is a known complication of AS; however, the best initial step in management is a radiograph of the lumbosacral spine to diagnose AS. Answer 3: MRI of the sacroiliac joint is the most accurate diagnostic test of AS. It would not be performed as the initial test. Answer 4: CT scan of the chest would be useful for assessing the lungs and rib cage for any cause of this patient's shortness of breath; however, it would not be the best initial test. Answer 5: Ultrasound could be an appropriate initial diagnostic test for a tension pneumothorax or to assess for an abdominal aortic aneurysm; however, it would not be appropriate in assessing AS. Bullet Summary: Ankylosing spondylitis presents with back pain that is worse in the morning and improves with activity, and the best initial test is a radiograph of the lumbosacral spine.
A 25-year-old man presents to his primary care physician with lower back pain. He states that he has had the pain for the past two years. The patient works as a butcher, and recently was moving heavy meat carcasses. The patient states that his pain is worse in the morning and that nothing improves it aside from swimming. The patient has a past medical history of anabolic steroid abuse, acne, hypertension and obesity. His current medications are hydrochlorothiazide, ibuprofen, topical benzoyl peroxide, and acetaminophen. On physical exam there is no tenderness upon palpation of the spine. There is limited range of motion of the spine in all 4 directions. Which of the following is most likely to confirm the most likely diagnosis in this patient? A) Radiograph of the spine B) MRI of the spine C) The straight leg test and the clinical presentation D) MRI of the sacroiliac joint E) HLA typing
This patient is a young male presenting with lower back pain that is worse in the morning and relieved with exercise (swimming). The most accurate test (and most sensitive test) for making the diagnosis is a MRI of the sacroiliac (SI) joint. Any young, male patient that presents with lower back pain that is worse with rest, and improves with exercise suggests a diagnosis of ankylosing spondylitis. Patients will often have a limited range of motion on physical exam. Initial tests include a radiograph of the spine (demonstrating a classic bamboo spine, Illustration A) and a radiograph of the sacroiliac joint. The most accurate test for diagnosis is MRI of the SI joint which is a very sensitive test and could demonstrate changes earlier than a radiograph of the spine. Illustration A demonstrates a radiograph of the spine with the classic "bamboo spine," seen in ankylosing spondylitis. Incorrect Answers: Answer 1: A radiograph of the spine would demonstrate a bamboo spine (seen in Figure A); however, it is not as sensitive or specific as a MRI of the SI joint. Answer 2: MRI of the spine would be appropriate to diagnose a herniated disc (pain/electricity that shoots down the legs) or spinal stenosis (pain in the back/legs that is positional). It is not the most accurate test for diagnosing ankylosing spondylitis. Answer 3: The straight leg test can be used to rule out or rule in a herniated disc (though the most accurate test would be a MRI of the spine). In the absence of radicular pain, physical exam is all that is needed to diagnose a muscle strain which is possible given this patients history of moving furniture. The patients pain that is relieved with activity does not support this diagnosis. Answer 5: HLA B-27 is not a specific test for diagnosing ankylosing spondylitis. Up to 8% of the population can have this variation and HLA typing can only help to confirm the diagnosis though it is not particularly accurate. Bullet Summary: Ankylosing spondylitis presents in young males with back pain and stiffness that is worse at rest and relieved with activity. The most accurate diagnostic test is a MRI of the SI joint and the treatment of choice is physical therapy followed by NSAIDS. In patients with an inadequate response to two consecutive NSAIDs, TNF inhibitors (i.e., infliximab, adalimumab) may be added or used as second-line treatment (note that steroids are not a part of treatment).
A 23-year-old man complains of lower back pain that began approximately 6 months ago. He is unsure why he is experiencing this pain and notices that this pain is worse in the morning after waking up and improves with physical activity. Ibuprofen provides significant relief. He denies bowel and bladder incontinence or erectile dysfunction. Physical exam is notable for decreased chest expansion, decreased spinal range of motion, 5/5 strength in both lower extremities, 2+ patellar reflexes bilaterally, and an absence of saddle anesthesia. Which of the following is the most appropriate next test for this patient? A) ESR B) HLA-B27 C) MRI sacroiliac joint D) Radiograph sacroiliac joint E) Slit-lamp examination
This patient is a young man with back pain that improves with exertion and has a decreased range of motion which is consistent with ankylosing spondylitis (AS). The most appropriate initial diagnostic test is a radiograph of the sacroiliac joint. AS is a seronegative spondyloarthropathy that commonly causes chronic inflammatory disease in the spine and sacroiliac joints. Typically, this spondyloarthropathy affects young men. AS involves the spine and sacroiliac joints, hips, shoulders, entheses, and peripheral joints, as well as having extra-articular manifestations (such as uveitis, inflammatory bowel disease, and aortic insufficiency). Radiographic findings include symmetric sacroilitis, ankyloses, and sacroiliac joint fusion. There is a strong correlation between HLA-B27 and AS, and HLA-testing is warranted in an individual with imaging suggestive of AS. Nonsteroidal anti-inflammatory drugs (NSAIDs) can provide substantial relief from inflammatory back pain. NSAIDs are considered first line in the pharmacologic treatment of AS. If patients are unresponsive to NSAIDs, sulfasalazine can then be used. The best initial test in the workup of AS is a radiograph of the sacroiliac joint and the most accurate test is an MRI of the sacroiliac joint. Incorrect Answers Answer 1: Erythrocyte sedimentation rate (ESR) is a nonspecific assessment of inflammation. It is positive in many infectious and inflammatory conditions but is nonspecific and would not change further imaging workup. Answer 2: HLA-B27 testing is not the best initial test. If the patient has a negative radiograph and has some findings suggestive of a spondyloarthropathy, HLA-B27 testing should be pursued. HLA-B27 is seen in PAIR conditions - psoriasis, ankylosing spondylitis, inflammatory bowel disease, and reactive arthritis. Answer 3: MRI sacroiliac joint is incorrect but may be pursued if radiography is inconclusive. This is the most accurate test in the workup of ankylosing spondylitis. Answer 5: Slit-lamp examination is incorrect as it would not confirm the diagnosis of AS. It is true that extra-articular manifestations of AS include uveitis which may warrant a slit-lamp exam. Bullet Summary: The most appropriate initial step in the workup of ankylosing spondylitis is a radiograph of the sacroiliac joint.
A 21-year-old male comes to your office complaining of blurry vision and eye pain. He said the pain started this morning in his right eye and has gotten progressively worse. He reports that the eye has become very sensitive to light and is "watery," which he feels is blurring his vision. The patient has no significant past medical history and takes no prescribed medications. A review of systems is notable only for frequent headaches and lower back pain. The headaches most often occur at night and are sometimes worse after he drinks alcohol. The back pain started a couple months ago, is worst when he wakes up, but improves after his daily morning run. The patient reports he drinks 3-4 beers per night on the weekends with friends. He smokes marijuana socially and at night to help him sleep. He is currently sexually active with multiple male sexual partners. An image of his eye upon examination is shown in Figure A. Which of the following is associated with this patient's condition? A) Ipsilateral rhinorrhea B) Recent episode of diarrhea C) Positive Rapid Plasma Reagin (RPR) test D) Rheumatoid factor positivity E) HLA-B27 positivity
This patient is presenting with a painful eye (uveitis), photophobia, and lower back pain that improves after exercise, suggesting the diagnosis of ankylosing spondylitis (AS). AS is strongly associated with HLA-B27 positivity. The classic presentation for AS is lower back pain or stiffness (lumbar region) that persists for over 3 months and pain that is worse in the morning, but improves with activity. Other sequelae include: 1. Restrictive-lung disease 2. Uveitis 3. Aortitis with aortic valve insufficiency that may progress to arrhythmia This patient is presenting with uveitis which often affects the anterior uveal tract and can develop into a hypopyon within the anterior chamber of the eye (Illustration A). The workup for this patient should include the best initial test of radiography demonstrating the classic "bamboo spine" appearance caused by vertical joint fusion and sclerotic changes in the sacroiliac joint which could be followed by the most accurate diagnostic test, MRI of the SI joint. Other lab findings in this patient would be an elevated ESR/CRP. Treatment involves indomethacin for pain management and TNF inhibitors or sulfasalazine for severe/refractory disease in addition to physical therapy. Figure A shows anterior uveitis with a hypopyon. Illustration A shows a slit lamp photograph of the eye with precipitates located inferiorly, associated with anterior uveitis. Incorrect Answers: Answer 1: Ipsilateral rhinorrhea, combined with nasal congestion, eye pain, and ptosis, is classically associated with cluster headaches. Cluster headaches are common in young men. Attacks occur at the same time, usually at night, can be precipitated by alcohol, and will last between 15-30 minutes. Treatment involves 100% oxygen for acute episodes and verapamil (calcium channel blocker) for prophylaxis. Answer 2: Diarrhea caused by GI infections with organisms such as Chlamydia, Campylobacter, and Yersinia can be associated with Reiter's syndrome or reactive arthritis. The classic triad of symptoms includes urethritis, conjunctivitis/uveitis, and arthritis. Diagnosis is by clinical presentation, but biomarkers may show elevations in ESR/CRP and 75% of patients will be HLA-B27 positive. The best initial step in management is to treat the underlying infection. Answer 3: A positive RPR is sensitive for syphilis. Secondary syphilis may be associated with a rash on the palms and soles, genital condylomata lata, headache, joint inflammation, inflammation of the optic nerve, and uveitis. 20-65% of men do not report having the classic primary syphilis chancre. Treatment involves benzathine penicillin G. Answer 4: Rheumatoid factor can be associated with many rheumatologic conditions including rheumatoid arthritis and juvenile idiopathic arthritis. Rheumatoid arthritis presents with symmetric arthritis in both small and large joints, most commonly the wrists, MCP and PIP joints, knees, and neck. Extra-articular joint manifestations can include subcutaneous nodules, vasculitis, pleural effusion, and keratoconjunctivitis sicca (dry eye). NSAIDS are indicated as first-line therapy. Bullet Summary: Ankylosing spondylitis often presents with lower back pain and stiffness in the morning, which improves with activity. It commonly occurs in young patients and is strongly associated with HLA-B27 positivity. Radiographic imaging demonstrates a "bamboo" spine and involvement of the sacroiliac joint while MRI of the SI joint is the most accurate diagnostic test.
A 45-year-old woman presents to her primary care physician for knee pain. She states that she has been experiencing a discomfort and pain in her left knee that lasts for several hours but tends to improve with use. She takes ibuprofen occasionally which has been minimally helpful. She states that this pain is making it difficult for her to work as a cashier. Her temperature is 98.6°F (37.0°C), blood pressure is 117/58 mmHg, pulse is 90/min, respirations are 14/min, and oxygen saturation is 97% on room air. Physical exam reveals a stable gait that the patient claims causes her pain. The patient has a non-pulsatile, non-erythematous, palpable mass over the posterior aspect of her left knee that is roughly 3 to 4 cm in diameter and is hypoechoic on ultrasound. Which of the following is associated with this patient's condition? A) Artery aneurysm B) Herniated nucleus pulposus C) Inflammation of the pes anserine bursa D) Type IV hypersensitivity E) Venous valve failure
This patient is presenting with a popliteal cyst (Baker cyst) which is associated with rheumatoid arthritis (a type IV hypersensitivity). A popliteal cyst presents with pain in the knee that is typically localized to the posterior aspect. A palpable mass is present in the medial and posterior aspect of the knee which represents herniated synovial fluid. This condition is associated with anything that damages the joint. This can include osteoarthrtis, rheumatoid arthritis, or trauma (in particular meniscal tears). Rheumatoid arthritis is considered a type IV hypersensitivity reaction. Incorrect Answers: Answer 1: Artery aneurysm describes a popliteal artery aneursym which is a possible cause of a mass in the posterior aspect of the knee. However, it would not present with chronic pain. This pathology is typically asymptomatic in its early stages. Answer 2: Herniated nucleus pulposus describes a herniated disc which would present with radicular symptoms such as pain that radiates down the leg. Answer 3: Inflammation of the pes anserine bursa describes pes anserine bursitis which presents with pain over the infero-medial aspect of the ventral knee. It is not associated with a popliteal cyst. Answer 5: Venous valve failure describes venous insufficiency which can present with tortuous dilated veins and stasis ulcers. It is not associated with a popliteal cyst. Bullet Summary: Popliteal cysts are associated with any process that damages the joint such as trauma, osteoarthritis, or rheumatoid arthritis.
A 40-year-old construction worker comes to your office complaining of lower back pain. The patient reports that the pain first started 6 months ago, and that he thought it was from lifting heavy objects at work. Initially his symptoms occurred only when he woke up and then slowly improved at the end of the workday, but now his symptoms last almost his entire shift. A review of systems is notable for fatigue, palpitations, and dyspnea. He notes that the fatigue has also been getting progressively worse. The patient's past medical history is significant for eczema and inflammatory bowel disease. His only medications are NSAIDs to help his lower back pain. He smokes cigars socially. On physical exam, you note mild bilateral lower extremity edema and restricted anterior flexion of the lower spine. Straight leg raises of both legs elicits no lower back pain. The neurological exam is otherwise unremarkable. His temperature is 99°F (37.2°C), blood pressure is 121/82 mmHg, pulse is 94/min, reapirations are 15/min, and oxygen saturation is 97% O2 on room air. An EKG is performed, as shown in Figure A. Which of the following results would you expect on this patient's pulmonary function test? A) Normal vital capacity, normal total lung capacity, normal FEV1/FVC ratio, low DLCO B) Normal vital capacity, normal total lung capacity, decreased FEV1/FVC ratio, low DLCO C) Normal vital capacity, normal total lung capacity, decreased FEV1/FVC ratio, high DLCO D) Decreased vital capacity, decreased total lung capacity, normal FEV1/FVC ratio, normal DLCO E) Decreased vital capacity, decreased total lung capacity, normal FEV1/FVC ratio, low DLCO
This patient is presenting with chronic lower back pain that is worse in the morning, but improves throughout the day, suggesting the diagnosis of ankylosing spondylitis (AS). AS can cause a restrictive respiratory pattern on pulmonary function tests. The classic presentation for AS is lower back pain or stiffness (lumbar region) that persists for over three months and pain that is worse in the morning but improves with activity. On physical exam, the patient may exhibit a positive Schober's test, which implies diminshed anterior spinal flexion. Other sequelae of AS include restrictive-lung disease, uveitis, aortitis with aortic valve insufficiency that may progress to arrhythmia, and inflammatory bowel disease. This patient's dyspnea is suspicious for restrictive lung-disease as a sequelae of AS, which is caused by fusion of the costovertebral joints and ankylosis of the thoracic spine, as shown in Illustration A. The anterior chest wall may also be involved, and the patient may experience pleuritic chest pain. The pattern for restrictive lung diseases on pulmonary function tests includes decreased vital capacity, decreased total lung capacity, but a normal FEV1/FVC ratio. A person with lung disease caused by AS would also likely have a normal DLCO, as there is no diffusion impairment. Figure A shows an EKG with normal sinus rhythm. Incorrect Answers: Answer 1: Normal vital capacity (VC), total lung capacity (TLC), and FEV1/FVC ratio, but a low DLCO indicates diminished diffusion without an obstructive or restrictive respiratory pattern. The differential for these results is extensive, including congestive heart failure, dermatomyositis, early interstitial lung disease, systemic lupus erythematosus, systemic sclerosis, and granulomatosis with polyangitis. Answer 2: Normal VC and TLC, but decreased FEV1/FVC ratio and a low DLCO indicates an obstructive respiratory pattern with diminished diffusion. The differential for these results includes cystic fibrosis and emphysema. Answer 3: Normal VC and TLC, but decreased FEV1/FVC ratio and a high DLCO indicates an obstructive respiratory pattern with increased diffusion. The differential for these results includes asthma, polycythemia, and pulmonary hemorrhage. Answer 5: Decreased VC and TLC, but normal or even increased FEV1/FVC ratio and low DLCO indicates a restrictive respiratory pattern with diminished diffusion. The differential for these results includes interstitial lung disease such as asbestosis, beryillosis, hypersensitivity pneumonitis, idiopathic pulmonary fibrosis, sarcoidosis, and later silicosis. Bullet Summary: One major sequalae of ankylosing spondylitis is restrictive lung disease which shows up on pulmonary function tests with decreased vital capacity and total lung capacity but a normal FEV1/FVC ratio and DLCO. Note Highlight (default)
A 44-year-old woman presents to the emergency department with pain in her mouth. She states that it started a few days ago and has been gradually worsening. The patient has experienced malaise and symmetric joint pain that is worse in the morning for the past several months and is currently being treated. She otherwise states that she feels well. Her temperature is 98.3°F (36.8°C), blood pressure is 124/85 mmHg, pulse is 88/min, respirations are 14/min, and oxygen saturation is 99% on room air. Physical exam is notable for the finding in Figure A. Laboratory studies are ordered as seen below. Serum: Na+: 141 mEq/L Cl-: 102 mEq/L K+: 4.3 mEq/L HCO3-: 25 mEq/L BUN: 20 mg/dL Glucose: 96 mg/dL Creatinine: 1.1 mg/dL Ca2+: 10.2 mg/dL AST: 62 U/L ALT: 90 U/L Which of the following is also likely to be found in this patient? A) Coronary artery aneurysm B) Hyperglycemia C) Lung malignancy D) Macrocytic anemia E) Nephritic syndrome
This patient is presenting with fatigue, hepatotoxicity, and mucositis/stomatitis in the setting of rheumatoid arthritis (symmetric joint pain in the morning that improves throughout the day) which is concerning for methotrexate toxicity. Macrocytic anemia is a possible complication secondary to methotrexate as well. Rheumatoid arthritis typically presents in middle aged women with symmetric joint pain that is typically worse in the morning and improves throughout the day. Other symptoms can include fatigue, malaise, weight loss, and anemia of chronic disease. There are many agents that can be used to treat rheumatoid arthritis. For treatment of minor symptoms, typically NSAIDs are all that are needed. However, for persistent or recurrent disease, disease modifying rheumatic drugs may be indicated with methotrexate being a common drug used. Methotrexate is a dihydrofolate reductase inhibitor and it is this antagonism of folate that can lead to a macrocytic anemia. Figure A is the physical exam finding of stomatitis/mucositis which can be seen with methotrexate use. Incorrect Answers: Answer 1: Coronary artery aneurysm could be seen in Kawasaki disease which is treated with aspirin and IVIG. It typically presents with CRASH and burn (Conjunctivitis, Rash, Adenopathy, Strawberry tongue, Hand swelling/desquamating rash, and fever (the burn)). Answer 2: Hyperglycemia is a side effect of steroids which are an appropriate medication for rheumatoid arthritis flares; however, it is not preferred long term and would not cause stomatitis/mucositis as a common side effect. Answer 3: Lung malignancy is a common complication of inhaling pollution or smoking; however, the common complication associated with methotrexate use is interstitial lung disease rather than causing an underlying malignancy. Answer 5: Nephritic syndrome or poststreptococcal glomerulonephritis presents with dark urine, periorbital edema, and other signs of fluid overload. It would occur after a streptococcal infection (such as impetigo or pharyngitis). Bullet Summary: Methotrexate can cause a macrocytic anemia.
A 54-year-old woman with a past medical history of mental retardation, hypertension, and diabetes presents to the emergency department with a change in her behavior. Her caretakers state that the patient's gait suddenly became ataxic, and she became less responsive than her normal non-verbal baseline. Her temperature is 98.5°F (36.9°C), blood pressure is 125/68 mmHg, pulse is 90/min, respirations are 15/min, and oxygen saturation is 99% on room air. Physical exam is notable for an unremarkable HEENT exam with normal facial features and no signs of airway compromise. Neurological exam is remarkable for new onset spasticity. The patient has 3+ reflexes and a positive Babinski sign. Musculoskeletal exam is only notable for symmetric swelling and deformities of the patient's hands bilaterally. Additionally, there is a "clunk" when posterior force is applied to the head while anterior force is applied to the cervical spine. Which of the following is the most likely risk factor that predisposed this patient to this condition? A) Cerebral palsy B) Diabetes mellitus C) Down syndrome D) Hypertension E) Rheumatoid arthritis
This patient is presenting with spasticity, hyperreflexia, a positive Babinski sign and other upper motor neuron findings in the setting of proximal and symmetric joint swelling suggesting a risk factor of rheumatoid arthritis as the underlying predisposing factor to her atlantoaxial joint instability. The atlantoaxial joint in the cervical spine is prone to instability by both degenerative and traumatic processes. Potential degenerative etiologies include Down syndrome, rheumatoid arthritis, and os odontoideum among many other possible etiologies. In contrast, traumatic fractures such as a type I odontoid fracture, an atlas fracture, and transverse ligament injuries are other etiologies of atlantoaxial joint instability. If there is impingement upon the spinal cord, patients may present with hyperreflexia, muscle weakness, a broad-based and ataxic gait (in particular if vertigo is present), decreased hand dexterity, a loss of motor milestones (in pediatric patients), and bladder problems. Additionally, the patient's neck "clunks" when posterior force is applied to the head while anterior force is applied to the cervical spine is representative of a positive "clunk" test and atlantoaxial instability. Radiography can confirm the diagnosis and treatment is based on the underlying etiology. In rheumatoid arthritis, degenerative changes can predispose patients to this condition and treatment can involve surgical spinal decompression. Incorrect Answers: Answer 1: Cerebral palsy is a risk factor for atlantoaxial subluxation; however, this is a diagnosis that would be present with the patient from birth. There are no symptoms suggestive of this diagnosis other than the patient's spasticity; however, this is new onset and is secondary to an acute change rather than a chronic disorder. Answer 2: Diabetes mellitus is a risk factor for many different pathologies and can cause microvascular damage increasing the risk of cardiovascular disease, peripheral arterial disease, and peripheral neuropathy. Answer 3: Down syndrome is a risk factor for atlantoaxial instability; however, this patient would have a history of mental retardation in addition to other finding such as a single palmar crease and facial abnormalities which were not noted in this patient. Answer 4: Hypertension can cause lipohyalinosis and hypertensive strokes which could present with focal neurological deficits. It is unlikely that a hypertensive stroke would present with upper motor neuron signs, spasticity, hyperreflexia, and a positive Babinski sign, in particular, in the setting of this patient's proximal and symmetric joint findings in her hands. Bullet Summary: Rheumatoid arthritis is a risk factor for atlantoaxial instability. NoteHighlight (default)
A 25-year-old man presents to his primary care physician for pain in his back. The patient describes the pain as feeling worse in the morning. He says it is a general stiffness that improves when he goes to the gym and lifts weights. He also states that his symptoms seem to improve when he leans forward or when he is cycling. The patient is a current smoker and is sexually active. He admits to having unprotected sex with many different partners this past year. The patient has no significant past medical history and is not currently taking any medications. On physical exam, the patient demonstrates notable kyphosis of the thoracic spine and decreased mobility of the back in all 4 directions. The patient's strength is 5/5 in his upper and lower extremities. The rest of his physical exam is within normal limits. Which of the following findings is associated with this patient's presentation? A) Decreased levels of IgA B) Diminished pulses in the lower extremity C) Narrowing of the spinal canal when standing upright D) Pain with elevation of his leg while laying down E) Punctate bleeding spots when dermatologic scales are removed
This patient is presenting with stiffness/pain in his back that is relieved with activity suggesting a diagnosis of ankylosing spondylitis. Ankylosing spondylitis is associated with psoriasis which can present with punctate bleeding spots when scales are removed (Auspitz sign). HLA-B27 is associated with PAIR - Psoriasis, Ankylosing spondylitis, Inflammatory bowel disease, and Reactive arthritis. When one of these conditions is diagnosed, it is important to consider determining if the patient is HLA-B27 positive and to work them up for these other conditions. Ankylosing spondylitis presents with back pain/stiffness in a young man that is relieved by activity. Psoraisis presents with silvery dermatologic scales that result in punctate bleeding when removed. It is possible for patients to have multiple PAIR disorders when they are HLA-B27 positive. Incorrect Answers: Answer 1: Decreased levels of IgA describes celiac disease which presents with diarrhea when the patient is exposed to gluten. HLA-B27 is associated with inflammatory bowel disease rather than celiac disease. Answer 2: Diminished pulses in the lower extremity would be found in vascular claudication. Vascular claudication occurs in older patients with atherosclerosis and presents with pain in the legs that is worsened by exertion and is relieved by rest. Answer 3: Narrowing of the spinal canal when standing upright is characteristic of neurogenic claudication which presents with back and leg pain that is worsened by standing upright and relieved by leaning forward. Answer 4: Pain with elevation of his leg while laying down would suggest a diagnosis of a herniated nucleus pulposus. This would classically present with pain that started after lifting a heavy object which resulted in electrical pain that radiates down the legs. Bullet Summary: HLA-B27 is a genetic risk factor for PAIR - Psoriasis, Ankylosing spondylitis, inflammatory bowel disease, and Reactive arthritis.
A 38-year-old woman presents to her primary care physician for a new patient appointment. She states that she feels well and has no current complaints. The patient recently started seeing a specialist for treatment for another medical condition but otherwise has had no medical problems. The patient lives alone and drinks 2 alcoholic beverages every night. She has had 3 sexual partners in her lifetime, uses oral contraceptive pills for contraception, and has never been pregnant. Physical exam reveals a pleasant, obese woman with normal S1 and S2 on cardiac exam. Musculoskeletal exam reveals swelling of the MCP and PIP joints of the hands as well as ulnar deviation of the fingers. Laboratory tests are ordered and results are below: Serum: Na+: 139 mEq/L Cl-: 100 mEq/L K+: 4.3 mEq/L HCO3-: 25 mEq/L BUN: 20 mg/dL Glucose: 99 mg/dL Creatinine: 1.1 mg/dL Ca2+: 10.2 mg/dL AST: 95 U/L ALT: 68 U/L Which of the following best explains this patient's abnormal laboratory values? A) Alcohol B) Bacterial infection C) Medication D) Obesity E) Viral infection
This patient is presenting with symptoms of rheumatoid arthritis (MCP/PIP swelling and ulnar deviation of the fingers) and recent treatment of it in the setting of elevated liver enzymes suggests that she was started on methotrexate which is a medication that commonly causes elevated liver enzymes. Methotrexate is used in malignancy, abortions, and rheumatologic diseases as a disease-modifying agent. Common side effects of methotrexate include gastrointestinal symptoms (such as nausea, vomiting, and diarrhea), bone marrow suppression, oral ulcers, and hepatotoxicity (increased AST/ALT). These patient should be routinely monitored for side-effects secondary to this medication. Incorrect Answers: Answer 1: Alcohol could cause an elevation of liver enzymes with AST > ALT; however, 2 glasses of alcohol per day is unlikely by itself to cause elevated liver enzymes. Answer 2: Bacterial infection could cause elevated liver enzymes; however, there is nothing in this patient's history to suggest a bacterial infection. Answer 4: Obesity could predispose this patient to non-alcoholic fatty liver disease; however, the onset of elevated liver enzymes after beginning treatment for rheumatoid arthritis supports the notion that this patient was started on methotrexate for her severe symptoms. Answer 5: Viral infection could represent hepatitis B or C; however, an acute viral infection would result in higher elevation of these enzymes and there are no risk factors in this patient's history that would support this diagnosis. Bullet Summary: Methotrexate has side effects of GI symptoms, oral ulcers, and hepatotoxicity.
A 29-year-old man presents to his primary care physician with complaints of pain in his joints. He states it started a few days ago and has persisted since. He states that he is otherwise healthy and is not currently taking any medications. The patient is in the military and is currently in basic training. Review of systems is notable for discomfort with urination. His temperature is 99.0°F (37.2°C), blood pressure is 133/90 mmHg, pulse is 90/min, respirations are 13/min, and oxygen saturation is 98% on room air. Physical exam is notable for the finding in Figure A. The knee and ankle joints demonstrate pain with passive range of motion, but there are no signs of erythema, edema, or crepitus. Which of the following is the most likely diagnosis? A) Osteoarthritis B) Reactive arthritis C) Rheumatoid arthritis D) Septic arthritis E) Stress fracture
This patient is presenting with urethritis (pain with urination), conjunctivitis, and asymmetric joint pain without an effusion concerning for reactive arthritis. Reactive arthritis is an autoimmune response to infection that classically involves a triad of noninfectious urethritis, arthritis, and conjunctivitis. This classic triad is actually only seen in approximately one-third of patients with reactive arthritis. The disease is associated with preceding gastrointestinal infection with organisms such as Salmonella, Shigella, Yersinia, or Campylobacter as well as preceding genitourinary infection with the most common causative organism being Chlamydia. Diagnosis is primarily based on history and clinical presentation. Treatment involves treating any underlying infections and NSAIDs for symptom control. Figure A is the physical exam finding of conjunctivitis. Incorrect Answers: Answer 1: Osteoarthritis presents with joint pain typically in an older/obese patient with a cold and painful joint. Pain is typically worse with exertion and relieved by rest. Answer 3: Rheumatoid arthritis typically presents in a middle-aged woman with symmetric arthralgias that are worse in the morning and improve throughout the day along with other systemic manifestations (such as malaise and low-grade fevers). Answer 4: Septic arthritis presents with a red, hot, inflamed, and exquisitely tender joint. It must be aspirated followed by antibiotics and surgical washout. Answer 5: Stress fracture presents with focal bone pain/tenderness in a patient who overexerts oneself (such as someone in the military in basic training). Radiographs may demonstrate a hair-line fracture or may be normal. Treatment involves rest and non-weight bearing status. Bullet Summary: Reactive arthritis presents with urethritis, uveitis, and arthritis.
A 32-year-old man presents to his physician with a complaint of pain with urination that has developed and persisted over the past 8 days. Upon awakening today, he also noted a clear discharge from his urethra. The patient states he is otherwise healthy. Social history is notable for the patient working at a local farm with livestock. Review of systems is notable for left knee and ankle pain for the past week and worsening of his seasonal allergies with red and itchy eyes. His temperature is 97.7°F (36.5°C), blood pressure is 122/83 mmHg, pulse is 89/min, respirations are 14/min, and oxygen saturation is 98% on room air. Which of the following is likely to be positive in this patient? A) Anti-CCP B) Anti-dsDNA C) HLA-B27 D) HLA-DR4 E) p-ANCA
This patient is suffering from a reactive arthritis given his joint pain, conjunctivitis (red and itchy eyes attributed to allergies), and urethritis (pain with urination and discharge), most likely precipitated by a Chlamydia infection. HLA-B27 positivity increases the risk of developing reactive arthritis. The triad of large joint oligoarthritis, urethritis, and uveitis/conjunctivitis characterizes reactive arthritis. Once arthritic symptoms have manifested, both microbial tests and blood/synovial fluid cultures are often negative, with serum antibodies as the only detectable abnormality. Reactive arthritis is associated with a Chlamydia infection; however, other infections including GI infections (from infectious causes such as Salmonella) can predispose patients to this condition. The diagnosis of reactive arthritis is primarily clinical; however, an elevated CRP and ESR in the setting of an HLA-B27 positive patient strengthens the diagnosis. HLA-B27 is commonly found in the PAIR conditions - psoriasis, ankylosing spondylitis, inflammatory bowel disease, and reactive arthritis. Incorrect Answers: Answers 1 & 4: Anti-CCP is a specific marker in rheumatoid arthritis and HLA-DR4 is associated with rheumatoid arthritis which presents in a middle-aged woman with symmetric joint pain that is worse in the morning. Associated joint deformity and other systemic symptoms can be seen. Answer 2: Anti-dsDNA is a specific marker for systemic lupus erythematosus which presents with a rash (malar or discoid), arthralgias, myalgias, kidney disease, hemolytic anemia, serositis, and many other systemic symptoms depending on the organ system affected. Answer 5: p-ANCA is positive in ulcerative colitis which presents with abdominal pain, fever, and grossly bloody stools with inflammation and symptoms limited to the colon (versus the entire bowel as seen in Crohn disease). Bullet Summary: HLA-B27 is associated with the PAIR diseases - psoriasis, ankylosing spondylitis, inflammatory bowel disease, and reactive arthritis.
A 24-year-old man presents to the emergency department after a motor vehicle accident. The patient was at a stop when he was rear-ended from behind by a vehicle traveling at 11 miles per hour. The patient complains of severe back pain but states he otherwise feels well. The patient is currently seeing a physical therapist who is giving him exercises to alleviate the back pain that is present every morning, relived by activity, and worse with inactivity. He is a student at the university and is struggling with his grades. His temperature is 98.4°F (36.9°C), blood pressure is 117/78 mmHg, pulse is 116/min, respirations are 12/min, and oxygen saturation is 99% on room air. Physical exam demonstrates a decreased range of motion of the patient's spine and tenderness to palpation over the vertebrae. The rest of the exam is deferred due to pain. The patient is requesting a note to excuse him from final exams and work. Which of the following is the most likely diagnosis in this patient? A) Herniated nucleus pulposus B) Malingering C) Musculoskeletal strain D) Spondylolisthesis E) Vertebral fracture
This patient with a history of ankylosing spondylitis (lower back pain in a young male that is worse in the morning, improve with activity, and is worse with rest) is presenting with severe back pain after trauma suggestive of a vertebral fracture. Ankylosing spondylitis typically presents in young males with lower back pain that is worse with rest and improves with activity. Patients will often exhibit a decreased range of motion on exam. The chronic inflammation of the spine in ankylosing spondylitis predisposes the patient to vertebral fractures even with minimal trauma. Patients will often also suffer from osteoporosis/osteopenia. Incorrect Answers: Answer 1: Herniated nucleus pulposus would present with radicular pain (electrical pain that shoots down the legs). Answer 2: Malingering would present with an inconsistent physical exam and a clear desire for secondary gain. Answer 3: Musculoskeletal strain is common in motor vehicle accidents; however, this low impact collision likely did not cause this. Answer 4: Spondylolisthesis presents with a palpable step off, pain, and sometimes neurological symptoms. Bullet Summary: Ankylosing spondylitis can cause inflammation and osteoporosis/osteopenia that predispose the patient to vertebral fractures.
A 57-year-old woman presents to her primary care physician with a concern for joint pain. She states that she often feels minor joint pain and morning stiffness in both of her hands every day, in particular in the joints of her fingers. Her symptoms tend to improve as the day goes on and she states they are not impacting the quality of her life. She lives alone as her partner recently died. She smokes 1 pack of cigarettes per day and drinks 2-3 alcoholic drinks per day. Her last menses was at the age of 45 and she works at a library. The patient has a history of diabetes and chronic kidney disease and her last GFR was 25 mL/min. Her temperature is 97.5°F (36.4°C), blood pressure is 117/58 mmHg, pulse is 90/min, respirations are 14/min, and oxygen saturation is 98% on room air. Physical exam is within normal limits. Which of the following interventions is appropriate management of future complications in this patient? A) Alendronate B) Ibuprofen C) Infliximab D) Methotrexate E) Prednisone
This patient with rheumatoid arthritis is smoking, post-menopausal, and sedentary, which are all risk factors for osteoporosis. Alendronate is a bisphosphonate that can be used as prophylaxis for osteoporosis. Rheumatoid arthritis typically presents with morning stiffness > 1 hour, symmetric joint swelling of the MCP and PIP joints, and systemic symptoms. Rheumatoid arthritis is associated with osteoporosis secondary to steroid use and inflammatory cytokines from the disease. Prophylactic measures include exercise, smoking cessation, and vitamin D and calcium supplementation. Alendronate is a bisphosphonate which inhibits osteoclasts and can help preserve bone density. Incorrect Answers: Answer 2: Ibuprofen could be indicated to manage this patient's symptoms; however, it would not reduce any long-term complications in this patient. NSAID use is associated with gastric ulcers and decreased renal function. Answer 3: Infliximab is a disease-modifying agent that could be used if methotrexate fails. It could increase the risk of infection secondary to immunosuppression. Answer 4: Methotrexate is a disease-modifying agent that could be indicated in severe disease and may prevent worsening of disease and permanent bony lesions. It would not be indicated in this patient with severe renal impairment. Answer 5: Prednisone could decrease this patient's symptoms; however, it would worsen the risk of osteoporosis in this patient. Steroids are also immunosuppressive and can worsen glycemic control. Bullet Summary: Rheumatoid arthritis is a risk factor for osteoporosis and prophylaxis includes bisphosphonates.
A 27-year-old male presents to his primary care physician with lower back pain. He notes that the pain started over a year ago but has become significantly worse over the past few months. The pain is most severe in the mornings. His past medical history is unremarkable except for a recent episode of right eye pain and blurry vision. Radiographs of the spine and pelvis show bilateral sacroiliitis. Which of the following is the most appropriate treatment for this patient? A) Indomethacin B) Methotrexate C) Cyclophosphamide D) Oral prednisone E) Bed rest
This patient's clinical presentation is consistent with ankylosing spondylitis (AS). The first-line treatment for pain in AS is indomethacin. AS is a seronegative (ANA and rheumatoid factor negative) spondyloarthropathy associated with the HLA-B27 allele. Patients typically present with back pain that is worse in the morning and may report a history consistent with anterior uveitis. On examination, patients usually exhibit a positive Schober's test, indicating restricted anterior flexion of the spine. Radiographs may show sclerosis in the sacroiliac joints as well as a "bamboo spine" appearance. Indomethacin is the treatment of choice for pain management, along with physical therapy. Refractory cases may require treatment with TNF-alpha inhibitors or sulfasalazine. Kataria and Brent review spondyloarthropathies. There are five categories of spondyloarthropathies: AS, psoriatic arthritis, reactive arthritis (such as Reiter's disease), inflammatory bowel disease associated arthritis, and undifferentiated spondyloarthritis. All spondyloarthropathies are associated with the HLA-B27 allele, and early treatment is recommended to slow symptom progression. Gorman et al. perform a randomized, double-blinded, placebo-controlled trial to evaluate the efficacy of TNF-alpha inhibitors for treatment of AS. Randomly assigned patients were treated with etanercept. The primary outcomes included decreased morning stiffness, joint swelling, and back pain. Over four months of therapy, they found a significant improvement of symptoms in patients who received treatment with etanercept compared to those who received placebo. This provided evidence that TNF-alpha inhibitors are useful in the treatment of AS. Illustration A shows the typical spinal changes that occur in AS. Illustration B is a radiograph with notable, bilateral sacroiliitis. Illustration C is a table showing the different characteristics of several spondyloarthropathies. Incorrect Answers: Answer 2: Methotrexate is a disease-modifying treatment commonly used for rheumatoid arthritis and sarcoidosis. Answer 3: Cyclophosphamide is used for treatment of refractory Sjogren's syndrome, scleroderma and ANCA-associated vasculitis. Answer 4: High-dose steroids are used for many inflammatory conditions, but are not used as first-line therapy for AS. Answer 5: Physical therapy, not bed rest, is recommended for patients with AS.
A 39-year-old woman presents to the emergency department with fever, cough, and shortness of breath. She reports developing flu-like symptoms 7 days ago but progressively worsened to the point where she experiences dyspnea on exertion. Her cough is accompanied by a mild amount of yellow sputum. Past medical history is notable for a previous admission to the hospital for pneumonia 4 months ago and an admission for bacteremia 6 weeks ago. She additionally has a history of IV heroin abuse, but her last use of heroin was 3 years ago. Temperature is 101.2°F (38.4°C), blood pressure is 104/70 mmHg, pulse is 102/min, and respirations are 20/min. Physical examination demonstrates coarse upper airway breath sounds over the right lower lung field. A faint 1/6 non-radiating systolic flow murmur is auscultated at the first right intercostal space. Abdominal examination is significant for moderate splenomegaly. Tenderness of the wrists and fingers is elicited on palpation, and range of motion is restricted. The patient comments that her range of motion and pain usually improve as the day goes on. Which of the following laboratory abnormalities is most likely to be found in this patient? A) Decreased anion gap B) Flow cytometry positive for CD11c and CD2 C) Leukocytosis with left-shift D) Neutropenia E) Positive HIV serology
This patient's presentation with wrist tenderness, decreased range of motion, and splenomegaly combined with a history of recurrent infections is consistent with a diagnosis of Felty syndrome, which will have neutropenia on laboratory analysis. Felty syndrome is an autoimmune disease characterized by a triad of rheumatoid arthritis (RA), splenomegaly, and neutropenia (< 1500/μL). Patients will present with recurrent severe infections accompanied by joint pain and will have splenomegaly on physical examination. Other laboratory findings include anemia of chronic disease and thrombocytopenia. Additionally, patients are usually positive for anti-CCP and/or rheumatoid factor, as the RA in Felty syndrome is usually seropositive and erosive. Incorrect Answers: Answer 1: Decreased anion gap may be caused by multiple myeloma, which can also cause immunosuppression and frequent infections. However, this patient's splenomegaly combined with finger and wrist pain with morning stiffness makes a diagnosis of Felty syndrome more likely. Answer 2: Flow cytometry positive for CD11c and CD2 is seen in hairy cell leukemia. Hairy cell leukemia can cause frequent infections and splenomegaly; however, the findings of wrist and finger pain with morning stiffness fit the clinical picture of RA, making a diagnosis of Felty syndrome more likely. Answer 3: Leukocytosis with a left shift is seen in the setting of infection. While this patient almost certainly has pneumonia, her history of frequent infections, splenomegaly, and stigmata of RA makes a diagnosis of Felty syndrome more likely. Answer 5: Positive HIV serology is diagnostic of HIV infection. While she does have a history of frequent infections and a history of IV drug abuse, her splenomegaly and stigmata of RA make a diagnosis of Felty syndrome more likely. Bullet Summary: Felty syndrome is a triad of rheumatoid arthritis, neutropenia, and splenomegaly.
A 67-year-old male with no significant medical history comes to your office complaining of pain and stiffness in his hands upon wakening in the morning. You perform an x-ray (Figure A). What is the most likely diagnosis? A) Gouty arthritis B) Osteoarthritis (OA) C) Rheumatoid arthritis (RA) D) Psoriatic arthritis E) Systemic lupus erythematosus (SLE)
This radiograph demonstrates complete loss of the articular cartilage at all four distal interphalangeal (DIP) joints, large osteophytes, and ankylosis of the DIP joint of the middle finger, making OA (answer 2) the most likely diagnosis. OA is a degenerative joint disease that typically occurs after age 40. The proximal interphalangeal joint (PIP)/DIP, foot, knee, and hip joints are commonly affected, while the metacarpophalangeal (MCP), shoulder, wrist, and elbow joints are rarely implicated. Symptoms include pain and stiffness that worsens with activity (and, if occurring upon awakening in the morning, lasts less than an hour). Classic radiographic findings include joint space narrowing, subchondral sclerosis, subchondral cysts, and osteophytes. While these clinical characteristics are more suggestive of OA over other arthritides, Peat et al. note that OA has no absolute clinical criteria. Thus, a clinical diagnosis of OA is supported by the presence of typical symptoms, physical examination, imaging features, and, occasionally, laboratory findings. As Altman et al. established, a classic method for diagnosing OA is the presence of pain plus several clinical characteristics including being greater than 50 years old, having morning stiffness for less than 30 minutes, crepitus on active motion (especially in the knee), bony tenderness, bony enlargement, and no palpable warmth. Figure A demonstrates the classic radiographic findings seen in OA. Incorrect Answers: Answer 1: Gouty arthritis is usually monoarticular and tophi may appear on imaging. Answer 3, 5: RA and SLE would present with constitutional symptoms and different radiographic findings. Answer 4: Psoriatic arthritis shows a pencil-in-a-cup deformity on imaging.
A 35-year-old woman presents to her family physician with a complaint of painful joints for the past 2 weeks. She reports symmetric bilateral joint pain in her hands, knees, and ankles. She has never had this before, and her past medical history is notable only for asthma. She states the pain is worse in the morning and improves throughout the day. Review of systems is notable for a recent low-grade fever with malaise. She works as a school teacher and is sexually active with men and women. Her temperature is 97.9°F (36.6°C), blood pressure is 120/84 mmHg, pulse is 70/min, respirations are 14/min, and oxygen saturation is 97% on room air. The patient is instructed to take ibuprofen and acetaminophen for her joint pain. She returns 1 month later stating that she has not needed to take the medications as her pain has been absent for the past 3 days. Which of the following is the most likely diagnosis?
[This patient's presentation of symmetric arthritis of the hands, knees, and ankles in the context of a preceding cold-like illness and symptoms that spontaneously resolve suggest a diagnosis of a parvovirus B19 infection. Arthritis is a relatively uncommon manifestation of parvovirus B19 infection in children; however, the majority of adults with the infection develop arthralgias and a nonspecific viral syndrome that self resolves. The distribution of the arthritis is symmetric (most commonly involving the PIP and MCP joints in addition to the knees, wrists, and ankles) and may resemble rheumatoid arthritis. The arthritis is often self resolves within weeks but may last for months to years in some instances. Pediatric manifestations of parvovirus may include erythema infectiosum (or fifth disease which presents with a "slapped-cheek" appearance and fever) and an aplastic crisis (in sickle cell disease or hereditary spherocytosis with severe anemia and a low reticulocyte count). Incorrect Answers: Answer 1: Osteoarthritis presents with joint pain typically in an overweight/elderly individual that is worse with exertion and that gets progressively worse in a weight-bearing joint. It does not spontaneously resolve, and it is not associated with a viral syndrome. Answer 3: Reactive arthritis (Reiter syndrome) is secondary to a bacterial infection (typically, Chlamydia) that commonly manifests with a triad of arthritis, conjunctivitis, and urethritis. Answer 4: Rheumatoid arthritis presents with symmetric joint pain that is worse in the morning and improves throughout the day. It is associated with joint deformity and is not associated with a preceding viral illness or spontaneous resolution. Answer 5: Transient synovitis typically occurs in pediatric patient and presents after a viral illness with hip pain that spontaneously resolves. Children can usually bear weight on the joint in this condition in contrast to septic arthritis. Bullet Summary: Parvovirus in adults is associated with a viral syndrome and arthralgias that are symmetric that resolve on its own.